insights ias | insightsonindia · 9/5/2018 · socialist ideas became even more popular ......

71
Insights IAS | InsightsonIndia www.insightsias.com 1 www.insightsonindia.com INSIGHTS IAS REVISION TESTS FOR UPSC CIVIL SERVICES PRELIMINARY EXAM – 2018 SERIES – 9 : EXPLAINATIONS (Days 33-36) 1. Which of the following statement is/are NOT correct ? 1. Karachi session is memorable for its resolution on Fundamental Rights and National Economic Programme. 2. The Communal Award 1932, provided for a separate electorate only for the depressed classes. Select the correct answer using the codes below. a) Only 1 b) Only 2 c) Both 1 and 2 d) Neither 1 nor 2 Solution: b Justification: Karachi session 1931 It was held to endorse the Gandhi-Irwin or Delhi Pact Reiterated the goal of Poorna Swaraj Resolution on Fundamental Rights and the National Economic Programme (Hence statement 1 is correct) Background Bhagat Singh, Sukhdev and Rajguru had been executed six days earlier. Even though Gandhiji had made every attempt to save their lives, there was anger among the people, especially the youth, as to why he had not refused to sign the Pact on this question. All along Gandhiji’s route to Karachi he was greeted with black flag demonstrations. The Congress passed a resolution drafted by Gandhiji by which it, ‘while dissociating itself from and disapproving of political violence in any shape or form,’ admired ‘the bravery and sacrifice’ of the three martyrs. Communal Award, 1932 expression of ‘Divide and Rule’ policy of British Award allotted to each minority a number of seats in the legislatures to be elected on the basis of a separate electorate that is Muslims would be elected only by Muslims and Sikhs only by Sikhs, and so on. Muslims, Sikhs and Christians had already been treated as

Upload: vandat

Post on 21-May-2018

214 views

Category:

Documents


0 download

TRANSCRIPT

Insights IAS | InsightsonIndia

www.insightsias.com 1 www.insightsonindia.com

INS

IGH

TS

IAS

RE

VIS

ION

TE

ST

S F

OR

UP

SC

CIV

IL S

ER

VIC

ES

PR

EL

IMIN

AR

Y E

XA

M – 2

01

8

SERIES – 9 : EXPLAINATIONS (Days 33-36)

1. Which of the following statement is/are NOT correct ?

1. Karachi session is memorable for its resolution on Fundamental Rights and National

Economic Programme.

2. The Communal Award 1932, provided for a separate electorate only for the depressed

classes.

Select the correct answer using the codes below.

a) Only 1

b) Only 2

c) Both 1 and 2

d) Neither 1 nor 2

Solution: b

Justification:

Karachi session 1931

It was held to endorse the Gandhi-Irwin or Delhi Pact

Reiterated the goal of Poorna Swaraj

Resolution on Fundamental Rights and the National Economic Programme

(Hence statement 1 is correct)

Background

Bhagat Singh, Sukhdev and Rajguru had been executed six days earlier. Even though

Gandhiji had made every attempt to save their lives, there was anger among the people,

especially the youth, as to why he had not refused to sign the Pact on this question. All

along Gandhiji’s route to Karachi he was greeted with black flag demonstrations.

The Congress passed a resolution drafted by Gandhiji by which it, ‘while dissociating itself

from and disapproving of political violence in any shape or form,’ admired ‘the bravery

and sacrifice’ of the three martyrs.

Communal Award, 1932

expression of ‘Divide and Rule’ policy of British

Award allotted to each minority a number of seats in the legislatures to be elected on the

basis of a separate electorate that is Muslims would be elected only by Muslims and Sikhs

only by Sikhs, and so on. Muslims, Sikhs and Christians had already been treated as

Insights IAS | InsightsonIndia

www.insightsias.com 2 www.insightsonindia.com

INS

IGH

TS

IAS

RE

VIS

ION

TE

ST

S F

OR

UP

SC

CIV

IL S

ER

VIC

ES

PR

EL

IMIN

AR

Y E

XA

M – 2

01

8

minorities. The Award declared the Depressed Classes (Scheduled Castes of today) also

to be a minority community entitled to separate electorate and thus separated them from

the rest of the Hindus.

(Hence statement 2 is incorrect)

Source: Bipan Chandra

2. Which of the following was/were factor/s responsible for the rise of Socialism in India ?

1. Russian Revolution of 1917

2. Disappointment with the Non-cooperation Movement

3. Economic Depression of 1929.

Select the correct answer using the codes below.

a) Only 1

b) Only 2

c) Only 1 and 3

d) 1, 2 and 3

Solution: d

Justification:

The impact of the Russian Revolution

It overthrew the despotic Czarist regime and declared the formation of the first socialist

state.

The new Soviet regime unilaterally renounced its imperialist rights in China and other parts

of Asia.

It showed if the common people — the workers and peasants and the intelligentsia — could

unite and overthrow the mighty Czarist empire and establish a social order where there was

no exploitation of one human being by another, then the Indian people battling against

British imperialism could also do so.

Socialist doctrines, especially Marxism, the guiding theory of the Bolshevik Party, acquired a

sudden attraction, especially for the people of Asia.

Socialist ideas became even more popular during the 1930s as the world was engulfed by the

great economic depression. Unemployment soared all over the capitalist world. The world

depression brought the capitalist system into disrepute and drew attention towards Marxism

and socialism.

Insights IAS | InsightsonIndia

www.insightsias.com 3 www.insightsonindia.com

INS

IGH

TS

IAS

RE

VIS

ION

TE

ST

S F

OR

UP

SC

CIV

IL S

ER

VIC

ES

PR

EL

IMIN

AR

Y E

XA

M – 2

01

8

Impact on India

Socialist ideas now began to spread rapidly especially because many young persons who had

participated actively in the Non- Cooperation Movement were unhappy with its outcome and

were dissatisfied with Gandhian policies and ideas as well as the alternative Swarajist

Several socialist and communist groups came into existence all over the country. In

Bombay, S.A. Dange published a pamphlet Gandhi and Lenin and started the first socialist

weekly, The Socialist;

Bengal, Muzaffar Ahmed brought out Navayug and later founded the Langal in cooperation

with the poet NazruI Islam

Punjab, Ghulam Hussain and others published Inquilab;

Madras, M. Singaravelu founded the Labour-Kisan Gazette.

Student and youth associations were organized all over the country from 1927 onwards.

Hundreds of youth conferences were organized all over the country during 1928 and 1929

with speakers advocating radical solutions for the political, economic and social ills from

which the country was suffering. Jawaharlal Nehru and Subhas Bose toured the country

attacking imperialism, capitalism, and landlordism and preaching the ideology of socialism.

The Revolutionary Terrorists led by Chandrasekhar Azad and Bhagat Singh also turned to

socialism.

Trade union and peasant movements grew rapidly throughout the 1920s.

Within the Congress the left-wing tendency found reflection in the election of Jawaharlal Nehru

as president for 1936 and 1937 and of Subhas Bose for 1938 and 1939 and in the formation of

the Congress Socialist Party. (Hence option d is correct)

Source: Bipan Chandra

3. Which of the following statements is NOT correct with respect to Government of India Act

1935?

a) It proposed for the establishment of an All India Federation

b) Representatives of the states to the federal legislature were to be directly elected by

the people.

c) It provided for the system of Provincial Autonomy

d) Governors appointed by the British Government retained special powers

Solution: b

Insights IAS | InsightsonIndia

www.insightsias.com 4 www.insightsonindia.com

INS

IGH

TS

IAS

RE

VIS

ION

TE

ST

S F

OR

UP

SC

CIV

IL S

ER

VIC

ES

PR

EL

IMIN

AR

Y E

XA

M – 2

01

8

Justification:

Government of India Act of 1935

Act provided for the establishment of an All-India Federation to be based on the union of the

British Indian provinces and Princely States. The representatives of the States to the federal

legislature were to be appointed directly by the Princes who were to be used to check and

counter the nationalists.

The franchise was limited to about one-sixth of the adults.

Defence and foreign affairs would remain outside the control of the federal legislature, while

the Viceroy would retain special control over other subjects.

The provinces were to be governed under a new system based on provincial autonomy under

which elected ministers controlled all provincial departments.

Once again, the Governors, appointed by the British Government, retained special powers.

They could veto legislative and administrative measures, especially those concerning

minorities, the rights of civil servants, law and order and British business interests.

(Hence option b is correct)

Source: Bipan Chandra

4. Which of the following was NOT undertaken under the Congress Rule in 1937?

a) All emergency powers acquired by the provincial governments were repealed

b) Bans on illegal political organizations like Hindustan Seva Dal were removed

c) The ban on communist party was not removed

d) It attempted a complete overhaul of the agrarian structure

Solution: d

Justification:

The commitment of the Congress to the defence and extension of civil liberties was as old as

the Congress itself, and it is hardly surprising, therefore, that the Congress Ministries

registered major achievements in this sphere.

All emergency powers acquired by the provincial governments during 1932, through Public

Safety Acts and the like, were repealed

bans on illegal political organizations such as the Hindustan Seva Dal and Youth Leagues and

on political books and journals were lifted. Though the ban on the Communist Party

remained, since it was imposed by the Central Government and could only be lifted on its

Insights IAS | InsightsonIndia

www.insightsias.com 5 www.insightsonindia.com

INS

IGH

TS

IAS

RE

VIS

ION

TE

ST

S F

OR

UP

SC

CIV

IL S

ER

VIC

ES

PR

EL

IMIN

AR

Y E

XA

M – 2

01

8

orders, the Communists could in effect now function freely and openly in the Congress

provinces.

All restrictions on the press were removed. Securities taken from newspapers and presses

were refunded and pending prosecutions were withdrawn. The blacklisting of newspapers

for purposes of government advertising was given up.

Confiscated arms were returned and forfeited arms licenses were restored.

The Congress could not attempt a complete overhaul of the agrarian structure by completely

eliminating the zamindari system. This, for following reasons

According to the constitutional structure of the 1935 Act, the provincial Ministries did not

have enough powers to do so.

They also suffered from an extreme lack of financial resources, for the lion’s share of India’s

revenues was appropriated by the Government of India.

The Congress Ministries could also not touch the existing administrative structure, whose

sanctity was guarded by the Viceroy’s and Governor’s powers.

What is more important, the strategy of class adjustment also forbade it. A multi-class

movement could develop only by balancing or adjusting various, mutually clashing class

interests.

(Hence option d is correct)

Source: Bipan Chandra

5. The peasant movements in the 1930s and 1940 s were influenced by ?

1. Great Depression of 1929

2. Civil Disobedience Movement

3. Formation of Congress Socialist Party

Select the correct answer using the codes below.

a) Only 1

b) Only 1 and 2

c) Only 1 and 3

d) 1, 2 and 3

Solution: d

Insights IAS | InsightsonIndia

www.insightsias.com 6 www.insightsonindia.com

INS

IGH

TS

IAS

RE

VIS

ION

TE

ST

S F

OR

UP

SC

CIV

IL S

ER

VIC

ES

PR

EL

IMIN

AR

Y E

XA

M – 2

01

8

Justification:

The 1930s bore witness to a new and nation-wide awakening of Indian peasants to their own

strength and capacity to organize for the betterment of their living conditions. This awakening

was largely a result of the combination of particular economic and political developments.

The great Depression that began to hit India from 1929-30

new phase of mass struggle launched by the Indian National Congress in 1930.

Effect of depression

It brought agricultural prices crashing down to half or less of their normal levels dealt a severe

blow to the already impoverished peasants burdened with high taxes and rents. The Government

was obdurate in refusing to scale down its own rates of taxation or in asking zamindars to bring

down their rents. The prices of manufactured goods, too, didn’t register comparable decreases.

All told, the peasants were placed in a situation where they had to continue to pay taxes, rents,

and debts at pre-Depression rates while their incomes continued to spiral steadily downward

Civil Disobedience Movement

was launched in this atmosphere of discontent in 1930, and in many parts of the country it soon

took on the form of a no-tax and no-rent campaign. Peasants, emboldened by the recent success

of the Bardoli Satyagraha (1928), joined the protest in large numbers. In Andhra, for example,

the political movement was soon enmeshed with the campaign against re-settlement that

threatened an increase in land revenue.

In U.P., no-revenue soon turned into no-rent 3nd the movement continued even during the

period of truce following the Gandhi-Irwin Pact. Gandhiji himself issued a manifesto to the U.P.

kisans asking them to pay only fifty per cent of the legal rent and get receipts for payment of the

full amount.

Peasants in Gujarat, especially in Surat and Kheda, refused to pay their taxes and went hijrat to

neighbouring Baroda territory to escape government repression. Their lands and movable

property were confiscated.

In Bihar and Bengal, powerful movements were launched against the hated chowkidara tax by

which villagers were made to pay for the upkeep of their own oppressors.

In Punjab, a no-revenue campaign was accompanied by the emergence of kisan sabhas that

demanded a reduction in land revenue and water-rates and the scaling down of debts.

Forest satyagrahas by which peasants, including tribals, defied the forest laws that prohibited

them from use of the forests were popular in Maharashtra, Bihar and the Central Provinces.

Anti-zamindari struggles emerged in Andhra, and the first target was the Venkatagiri zamindari,

in Nellore district.

Insights IAS | InsightsonIndia

www.insightsias.com 7 www.insightsonindia.com

INS

IGH

TS

IAS

RE

VIS

ION

TE

ST

S F

OR

UP

SC

CIV

IL S

ER

VIC

ES

PR

EL

IMIN

AR

Y E

XA

M – 2

01

8

Also, in 1934, with the formation of the Congress Socialist Party (CSP). the process of the

consolidation of the Left forces received a significant push forward. The Communists, too, got

the opportunity, by becoming members of the CSP to work in an open and legal fashion. This

consolidation of the Left acted as a spur to the formation of an all-India body to coordinate the

kisan movement, a process that was already under way through the efforts of N.G. Ranga and

other kisan leaders.

(Hence option d is correct)

Source: Bipan Chandra

6. Which of the following is NOT correct with respect to the freedom struggle in Princely India?

a) Until Quit India movement the Congress did not initiate any political activity in the states.

b) All India states people’s conference was organized in December 1927.

c) Praja Mandals were formed due to the influence of Civil Disobedience Movement.

d) At Tripuri Congress in 1939 Congress decided to involve directly in the political activity

of princely states

Solution: c

Justification:

The advance of the national movement in British India, and the accompanying increase in

political consciousness about democracy, responsible government and civil liberties had an

inevitable impact on the people of the States.

In the first and second decade of the twentieth century, runaway terrorists from British India

seeking shelter in the States became agents of politicization.

A much more powerful influence was exercised by the Non-Cooperation and Khilafat Movement

launched in 1920; around this time and under its impact, numerous local organizations of the

States’ people came into existence. Some of the States in which praja mandals or States’ People’s

Conferences were organized were Mysore, Hyderabad, Baroda, the Kathiawad States, the Deccan

States, Jamnagar, Indore, and Nawanagar.

This process came to a head in December 1927 with the convening of the All India States’

People’s Conference (AISPC) which was attended by 700 political workers from the States. The

men chiefly responsible for this initiative were Baiwantrai Mehta, Manikial Kothari and G.R.

Abhayankar.

Insights IAS | InsightsonIndia

www.insightsias.com 8 www.insightsonindia.com

INS

IGH

TS

IAS

RE

VIS

ION

TE

ST

S F

OR

UP

SC

CIV

IL S

ER

VIC

ES

PR

EL

IMIN

AR

Y E

XA

M – 2

01

8

Indian national congress

1) The policy of the INC towards the Indian states had been first enunciated in 1920 at Nagpur

when a resolution calling upon the Princes to grant full responsible government in their States

had been passed.

2) Simultaneously, however, the Congress, while allowing residents of the States to become

members of the Congress, made it clear that they could not initiate political activity in the States

in the name of Congress but only in their individual capacity or as members of the local political

organizations.

3) Given the great differences in the political conditions between British India and the States,

and between the different States themselves, the general lack of civil liberties including freedom

of association, the comparative political backwardness of the people, and the fact that the Indian

States were legally independent entities, these were understandable restraints imposed in the

interest of the movements in the States as ell as the movement in British India.

4) The main emphasis was that people of the States should build up their own strength and their

capacity to struggle for their demands.

5) In 1927, the Congress reiterated as resolution of 1920, and in 1929. Jawaharlal Nehru, in his

presidential address to the famous Lahore Congress, declared that ‘the Indian states cannot live

apart from the rest of India. The only people who have a right to determine the future of the

states must be the people of those states’) In later years, the Congress demanded that the Princes

guarantee fundamental rights to their people.

In the mid thirties, two associated developments brought about a distinct change in the situation

in the Indian States.

Government of India Act of 1935 projected a scheme of federation in which the Indian States

were to be brought into a direct constitutional relationship with British India and the States were

to send representatives to the Federal Legislature. The catch was that these representatives

would be nominees of the Princes and not democratically elected representatives of the people.

They would number one-third of the total numbers of the Federal legislature and act as a solid

conservative block that could be trusted to thwart nationalist pressures. The Indian National

Congress and the AISPC and other organizations of the States’ people clearly saw through this

imperialist maneuver and demanded that the States be represented not by the Princes’ nominees

but by elected representatives of the people. This lent a great sense of urgency to the demand

for responsible democratic government in the States.

assumption of office by Congress Ministries in the majority of the provinces in British India in

1937. The fact that the Congress was in power created a new sense of confidence and expectation

in the people of the Indian States and acted as a spur to greater political activity. The Princes too

had to reckon with a new political reality — the Congress was no longer just a party in opposition

but a party in power with a capacity to influence developments in contiguous Indian States.

The years 1938-39, in fact, stand out as years of a new awakening in the Indian States and were

witness to a large number of movements demanding responsible government and other reforms.

Praja mandals mushroomed in many States that had earlier no such organizations. Major

Insights IAS | InsightsonIndia

www.insightsias.com 9 www.insightsonindia.com

INS

IGH

TS

IAS

RE

VIS

ION

TE

ST

S F

OR

UP

SC

CIV

IL S

ER

VIC

ES

PR

EL

IMIN

AR

Y E

XA

M – 2

01

8

struggles broke out in Jaipur, Kashmir, Rajkot, Patiala, Hyderabad, Mysore, Travancore, and the

Orissa States.

These new developments brought about a significant change in Congress policy as well.

Whereas, even in the Haripura session in 1938, the Congress had reiterated its policy that

movements in the States should not be launched in the name of the Congress but should rely on

their own independent strength and fight through local organizations, a few months later, on

seeing the new spirit that was abroad among the people and their capacity to struggle. Gandhiji

and the Congress changed their attitude on this question. The radicals and socialists in the

Congress, as well as political workers in the States, had in any case been pressing for this change

for quite some time.

Following upon this, the Congress at Tripuri in March 1939 passed a resolution enunciating its

new policy: ‘The great awakening that is taking place among the people of the States may lead to

a relaxation, or to a complete removal of the restraint which the Congress imposed upon itself,

thus resulting in an ever increasing identification of the Congress with the States’ peoples’.3 Also

in 1939, the AISPC elected Jawaharlal Nehru as its President for the Ludhiana session, thus

setting the seal on the fusion of the movements in Princely India and British India. The outbreak

of the Second World War brought about a distinct change in the political atmosphere.

(Hence option c is correct)

Source: Bipan Chandra

7. Which of the following statements is/are correct about the capitalist class in India during the

freedom struggle ?

1. Indian capitalist class grew from about the mid-19th century with largely an independent

capital base and not as junior partners of foreign capital.

2. On the whole it was not tied up in a subservient position with pro-imperialist feudal

interests either economically or politically.

Select the correct answer using the codes below.

a) Only 1

b) Only 2

c) Both 1 and 2

d) Neither 1 nor 2

Solution: c

Justification:

The economic development of the Indian capitalist class in the colonial period was substantial

and in many ways the nature of its growth was quite different from the usual experience in other

Insights IAS | InsightsonIndia

www.insightsias.com 10 www.insightsonindia.com

INS

IGH

TS

IAS

RE

VIS

ION

TE

ST

S F

OR

UP

SC

CIV

IL S

ER

VIC

ES

PR

EL

IMIN

AR

Y E

XA

M – 2

01

8

colonial countries. This had important implications regarding the class’s position vis-a-vis

imperialism.

First, the Indian capitalist class grew from about the mid 19th century with largely an

independent capital base and not as junior partners of foreign capital or as compradors. Hence

statement1 is correct.

Second, the capitalist class on the whole was not tied up in a subservient position with pro-

imperialist feudal interests either economically or politically. (Hence statement 2 is correct. )In

fact, a wide cross section of the leaders of the capitalist class actually argued, m 1944-45, in their

famous Bombay plan (the signatories to which were Purshottamdas Thakurdas, J.R.D. Tata, G.D.

Birla, Ardeshir Dalal, Sri Ram, Kasturbhai Lalbhai, A.D. Shroff and John Mathai) for

comprehensive land reform, including cooperativization of production, finance and marketing.’

(Hence option c is correct)

Source: Bipan Chandra

8. Which of the following is NOT correct with respect to Cripps Mission in India?

a) It was sent to seek active co-operations of Indians in World War II

b) It promised India complete Independence and a constitution making body

c) Any province which was not prepared to accept the new constitution would have the right

to sign a separate agreement with Britain regarding future status

d) For the present, British would continue to exercise sole control over defence of the

country

Solution: b

Justification:

Why cripps mission was sent ?

As the war situation worsened, President Roosevelt of the USA and President Chiang Kai-Shek

of China as also the Labour Party leaders of Britain put pressure on Churchill to seek the active

cooperation of Indians in the War. To secure this cooperation the British Government sent to

India in March 1942 a mission headed by a Cabinet minister Stafford Cripps, a leftwing Labourite

who had earlier actively supported the Indian national movement.

Cripps Mission

Even though Cripps announced that the aim of British policy in India was ‘the earliest possible

realization of self- government in India,’ the Draft Declaration he brought with him was

disappointing. The Declaration promised India

Insights IAS | InsightsonIndia

www.insightsias.com 11 www.insightsonindia.com

INS

IGH

TS

IAS

RE

VIS

ION

TE

ST

S F

OR

UP

SC

CIV

IL S

ER

VIC

ES

PR

EL

IMIN

AR

Y E

XA

M – 2

01

8

Dominion Status

constitution-making body after the War whose members would be elected by the provincial

assemblies and nominated by the rulers in case of the princely states.

The Pakistan demand was accommodated by the provision that any province which was not

prepared to accept the new constitution would have the right to sign a separate agreement with

Britain regarding its future status

For the present the British would continue to exercise sole control over the defence of the

country. Amery, the Secretary of State, described the Declaration as in essence a conservative,

reactionary and limited offer. Nehru, a friend of Cripps, was to write later: When I read those

proposals for the first time I was profoundly depressed.’

(Hence option b is correct)

Source: Bipan Chandra

9. Which of the following is NOT correct with respect to Cabinet Mission of 1946 ?

1. It was convinced that united India was unviable and hence accommodate the idea of

Pakistan

2. It proposed a center controlling defence, foreign affairs and Pakistan.

Select the correct answer using the codes below.

a) Only 1

b) Only 2

c) Both 1 and 2

d) Neither 1 nor 2

Solution: a

Justification:

Cabinet Mission

It was convinced that Pakistan was not viable and that the minorities’ autonomy must somehow

be safeguarded within the framework of a united India. (Hence statement 1 is incorrect).

The Mission Plan conceived three sections

A — comprising Madras, Bombay, Uttar Pradesh, Bihar, C.P. and Orissa;

B — consisting of Punjab, NWFP and Sind;

C — of Bengal and Assam

Insights IAS | InsightsonIndia

www.insightsias.com 12 www.insightsonindia.com

INS

IGH

TS

IAS

RE

VIS

ION

TE

ST

S F

OR

UP

SC

CIV

IL S

ER

VIC

ES

PR

EL

IMIN

AR

Y E

XA

M – 2

01

8

These sections would meet separately to decide on group constitutions.

There would be a common center controlling defence, foreign affairs and communications. After

the first general elections a province could come out of a group. After ten years a province could

call for a reconsideration of the group or union constitution.

Congress Reaction

It wanted that a province need not wait till the first elections to leave a group, it should have the

option not to join it in the first place. It had Congress- ruled provinces of Assam and NWFP

(which were in Sections C and B respectively) in mind when it raised this question.

Muslim League — wanted provinces to have the right to question the union constitution now,

not wait for ten years.

There was obviously a problem in that the Mission Plan was ambivalent on whether grouping

was compulsory or optional. It declared that grouping was optional but sections were

compulsory. This was a contradiction, which rather than removing, the Mission deliberately

quibbled about in the hope of somehow reconciling the irreconcilable.

(Hence option a is correct)

Source: Bipan Chandra

10. Which of the following pairs is NOT correctly matched?

Author Sanskrit work under Rashtrakuta

a) Trivikrama Nalachampu

b) Halayudha Kavirahasya

c) Gunabhadra Amogavritti

d) Viracharya Ganitasaram

Solution: c

Justification:

Sanskrit literature during Rashtrakutas

Trivikrama — Nalachampu

Halayudha — Kavirahasya (during the reign of Krishna III)

Insights IAS | InsightsonIndia

www.insightsias.com 13 www.insightsonindia.com

INS

IGH

TS

IAS

RE

VIS

ION

TE

ST

S F

OR

UP

SC

CIV

IL S

ER

VIC

ES

PR

EL

IMIN

AR

Y E

XA

M – 2

01

8

Jain literature under Rashtrakutas

Amogavarsha I, who was a Jain patronized many Jain scholars.

Jinasena — Parsvabhudaya (biography of Parsva)

Gunabhadra — Adipurana (life stories of various Jain saints)

Sakatayana — Amogavritti(grammer work)

Viracharya (mathematician) – Ganitasaram

Kannada literature during Rashtrakutas

Amogavarsha — Kavirajamarga (first poetic work in Kannada language)

Pampa – Vikramasenavijaya

Ponna — Santipurana.

(Hence option c is correct)

Source: Tamil Nadu History Class 11.

11. Which of the following statements is/are correct ?

1. Angkorwat temple is a Shaivite temple.

2. It is constructed in Dravidian style.

Select the correct answer using the codes below.

a) Only 1

b) Only 2

c) Both 1 and 2

d) Neither 1 nor 2

Solution: b

Justification:

Cambodia was colonised by Indians in the first century A.D. They influenced the native people

called the Khemers. The ruling dynasty was known as Kambojas and their country was Kamboja

or modern Cambodia. Under the early rulers Saivism and Vaishnavism made steady progress.

The Kamboja Empire at its greatest extent included Laos, Siam, part of Burma and the Malay

Peninsula. Numerous Sanskrit inscriptions give us a detailed history of its kings. A number of

Insights IAS | InsightsonIndia

www.insightsias.com 14 www.insightsonindia.com

INS

IGH

TS

IAS

RE

VIS

ION

TE

ST

S F

OR

UP

SC

CIV

IL S

ER

VIC

ES

PR

EL

IMIN

AR

Y E

XA

M – 2

01

8

Hindu literary works like the Vedas, the Ramayana, the Mahabharata, Panini’s grammar, Hindu

philosophical treatises were all known to the people of Cambodia. Like the Pallava kings, they

were called Varmans. Yasovarman and Suryavarman II were two well-known rulers.

Temples were built in South Indian style. There are plenty of Sanskrit inscriptions. The most

famous of these temples was the temple (wat) of Vishnu built by Suryavarman II in his capital

city Angkor. (Hence statement 1 is incorrect). It was popularly called as the Angkorwat Temple.

It is standing on top of a terraced structure. Each terrace is a sort of a covered gallery which

contains numerous relief sculptures. The temple is constructed on the Dravidian style (Hence

statement 2 is correct) and the sculptures depict episodes from the Ramayana and the

Mahabharata. The Kambhoja kingdom declined only in fifteenth century.

(Hence option b is correct)

Source: Tamil Nadu History Class 11.

12. Under the rule of Delhi Sultanate technology, who was the first ruler to establish a separate

military department called diwan-i-arz ?

a) Alauddin Khilji

b) Ghiyasuddin Balban

c) Iltutumish

d) Muhammad bin Tughlak

Solution: b

Justification:

Balban was determined to break the power of the Forty, the Turkish nobles. He spared only the

most obedient nobles and eliminated all others by fair or foul means. Malik Baqbaq, the governor

of Badaun, was publicly flogged for his cruelty towards his servants. Haybat Khan, the governor

of Oudh, was also punished for killing a man who was drunk. Sher Khan, the governor of Bhatinda

was poisoned. Instead of expanding his kingdom, Balban paid more attention to the restoration

of law and order. He established a separate military department – diwan-i-arz – and reorganized

the army. The outskirts of Delhi were often plundered by the Mewatis. Balban took severe action

against them and prevented such robberies. Robbers were mercilessly pursued and put to death.

As a result, the roads became safe for travel.

(Hence option b is correct.)

Source: Tamil Nadu History Class 11.

Insights IAS | InsightsonIndia

www.insightsias.com 15 www.insightsonindia.com

INS

IGH

TS

IAS

RE

VIS

ION

TE

ST

S F

OR

UP

SC

CIV

IL S

ER

VIC

ES

PR

EL

IMIN

AR

Y E

XA

M – 2

01

8

13. Which of the following is/are the contribution/s of Muhammad bin Tughlak ?

1. Introduction of Takkavi

2. Establishment of a separate department for agriculture.

Select the correct answer using the codes below.

a) Only 1

b) Only 2

c) Both 1 and 2

d) Neither 1 nor 2

Solution: c

Justification:

Muhammad bin Tughlaq

He increased the land revenue on the farmers of Doab (land between Ganges and Yamuna rivers)

in order to overcome financial difficulties. A severe famine was also ravaging that region at that

time. It had resulted in a serious peasant revolts. They fled from the villages but Muhammad bin

Tughlaq took harsh measures to capture and punish them. The revolts were crushed. However,

the Sultan realized later that adequate relief measures and the promotion of agriculture were

the real solution to the problem. Hence

He launched a scheme by which takkavi loans (loans for cultivation) were given to the

farmers to buy seed and to extend cultivation.

A separate department for agriculture, Diwan- i- Kohi was established.

Model farm under the state was created in an area of 64 square miles for which the

government spent seventy lakh tankas. This experiment was further continued by Firoz

Tughlaq.

(Hence option c is correct)

Source: Tamil Nadu History Class 11.

14. Which of the following is NOT correct about Firoz Shah Tughlak ?

a) He imposed Jiziya only on the non-Muslims

b) He developed royal factories called karkhanas

c) He abolished the iqtadari

d) He created a new department called Diwan-i-khairat

Insights IAS | InsightsonIndia

www.insightsias.com 16 www.insightsonindia.com

INS

IGH

TS

IAS

RE

VIS

ION

TE

ST

S F

OR

UP

SC

CIV

IL S

ER

VIC

ES

PR

EL

IMIN

AR

Y E

XA

M – 2

01

8

Solution: c

Justification:

Firoz Tughlaq

He strictly followed the advice of the ulemas in running the administration.

He pleased the nobles and assured hereditary succession to their properties. Thus the

iqta system was not only revived but also it was made hereditary.

As per the Islamic law he levied the taxes.

Jiziya was strictly imposed on non-Muslims.

first Sultan to impose irrigation tax. But at the same time he dug irrigation canals and

wells. The longest canal was about 200 kilometres from Sutlej to Hansi. Another canal

was between Yamuna and Hissar.

The special tax on 28 items was abolished by him since they were against the Islamic law.

He also developed royal factories called karkhanas in which thousands of slaves were

employed.

About 300 new towns were built during his reign. The famous among them was Firozabad

near Red Fort in Delhi, now called Firoz Shah Kotla.

Old monuments like Jama Masjid and Qutb-Minar were also repaired.

A new department called Diwan-i-Khairat was created to take care of orphans and

widows.

Free hospitals and marriage bureaus for poor Muslims were also established.

Firoz patronized scholars like Barani and Afif.

As he was guided by the ulemas, he was intolerant towards Shia Muslims and Sufis. He

treated Hindus as second grade citizens and imposed Jiziya. In this respect he was the

precursor of Sikandar Lodi and Aurangazeb.

Also he increased the number of slaves by capturing the defeated soldiers and young

persons.

(Hence option c is correct)

Source: Tamil Nadu History Class 11

Insights IAS | InsightsonIndia

www.insightsias.com 17 www.insightsonindia.com

INS

IGH

TS

IAS

RE

VIS

ION

TE

ST

S F

OR

UP

SC

CIV

IL S

ER

VIC

ES

PR

EL

IMIN

AR

Y E

XA

M – 2

01

8

15. What does khalisa land under the Delhi Sultanate refer to ?

a) Land assigned to officials

b) Land and revenue under the direct control of Sultans to service the royal court

c) Land granted to religious leaders

d) Land assigned to the nobility

Solution: b

Justification:

After consolidating their position in India, the Delhi Sultans introduced reforms in the land

revenue administration. The lands were classified into three categories:

iqta land – lands assigned to officials as iqtas instead of payment for their

khalisa land – land under the direct control of the Sultan and the revenues collected were

spent for the maintenance of royal court and royal household.

inam land – land assigned or granted to religious leaders or religious institutions.

(Hence option b is correct)

Source: Tamil Nadu History Class 11

16. Who authored the book Tabaqat – i- Nasari which deals with the general history of Muslim

dynasties upto 1260 ?

a) Ziauddin Barani

b) Alberuni

c) Hasan Nizami

d) Minhaj-us-Siraj

Solution: d

Justification:

The Delhi Sultans patronized learning and literature. Many of them had great love for Arabic and

Persian literature. Learned men came from Persia and Persian language got encouragement

Insights IAS | InsightsonIndia

www.insightsias.com 18 www.insightsonindia.com

INS

IGH

TS

IAS

RE

VIS

ION

TE

ST

S F

OR

UP

SC

CIV

IL S

ER

VIC

ES

PR

EL

IMIN

AR

Y E

XA

M – 2

01

8

from the rulers. Besides theology and poetry, the writing of history was also encouraged. Some

of the Sultans had their own court historians.

The most famous historians of this period were Hasan Nizami, Minhaj-us-Siraj, Ziauddin Barani,

and Shams-Siraj Afif

Barani’s Tarikhi- Firoz Shahi contains the history of Tughlaq dynasty

Minhaj-us-Siraj wrote Tabaqat-i- Nasari, a general history of Muslim dynasties up to 1260.

(Hence option d is correct)

Source: Tamil Nadu History Class 11

17. Consider the following statements

1. Theosophical Society was founded in Ireland in 1875.

2. It accepted the Hindu beliefs of reincarnation and karma.

Which of the above statements is/are correct ?

a) Only 1

b) Only 2

c) Both 1 and 2

d) Neither 1 nor 2

Solution: b

Justification:

Theosophical Society

founded in United States in 1875. In 1882, the headquarters was shifted to Adayar, on the

outskirts of Madras. (Hence statement 1 is incorrect).

Founders — Madame H.P. Blavatsky (1831-1891) and Colonel M.S. They were inspired by Indian

thought and culture.

The society believed that a special relationship could be established between a person’s soul and

God through contemplation, prayer, revelation, etc.

It accepted the Hindu beliefs in reincarnation and karma, and drew inspiration from the

philosophy of the Upanishads and samkhya, yoga and Vedanta schools of thought. (Hence

statement 2 is correct).

(Hence option b is correct)

Source: Spectrum

Insights IAS | InsightsonIndia

www.insightsias.com 19 www.insightsonindia.com

INS

IGH

TS

IAS

RE

VIS

ION

TE

ST

S F

OR

UP

SC

CIV

IL S

ER

VIC

ES

PR

EL

IMIN

AR

Y E

XA

M – 2

01

8

18. Which of the following is/ are correct about vesara architecture ?

1. It was initiated by Pulakesin I.

2. The earliest example of it is the Ravan Phadi cave.

3. Its hallmark is hybridization.

Select the correct answer using the codes below.

a) Only 1 and 3

b) Only 3

c) 1, 2 and 3

d) Only 2 and 3.

Solution: c

Justification:

vesara architecture – developed in southern part of the Deccan, i.e., in the region of Karnataka.

It’s hybrid of Nagara and Dravida style of temple architecture. (Hence statement 3 is correct).

Pulakesin I established the early western Chalukya kingdom when he secured the land around

Badami in 543. The early western Chalukyas ruled most of the Deccan till the mid-eighth century

when they were superseded by the Rashtrakutas.

Early Chalukyan activity also takes the form of rock-cut caves while later activity is of structural

temples. The earliest is probably the Ravana Phadi cave at Aihole which is known for its

distinctive sculptural style. One of the most important sculptures at the site is of Nataraja,

surrounded by larger-than-life-size depictions of the saptamatrikas: three to Shiva’s left and four

to his right. The figures are characterised by graceful, slim bodies, long, oval faces topped with

extremely tall cylindrical crowns and shown to wear short dhotis marked by fine incised

striations indicating pleating. They are distinctly different from contemporary western Deccan

or Vakataka styles seen at places such as Paunar and Ramtek. The hybridisation and

incorporation of several styles was the hallmark of Chalukyan buildings.

(Hence statement 1 and 2 are correct).

(Hence option c is correct)

Source: NCERT Class 11-An introduction to Indian Art part 1.

Insights IAS | InsightsonIndia

www.insightsias.com 20 www.insightsonindia.com

INS

IGH

TS

IAS

RE

VIS

ION

TE

ST

S F

OR

UP

SC

CIV

IL S

ER

VIC

ES

PR

EL

IMIN

AR

Y E

XA

M – 2

01

8

19. Which of the following is true about Padmavat ?

1. It is a Masnavi.

2. It was originally written in the Persian Nastaliq

Select the correct answer using the codes below.

a) Only 1

b) Only 2

c) Both 1 and 2

d) Neither 1 nor 2

Solution: c

Justification:

Padmavat (or Padmawat)

1) It’s an epic poem written in 1540 by Sufi poet Malik Muhammad Jayasi, who wrote it in the

Hindustani language of Awadhi, and originally in the Persian Nastaʿlīq script. (Hence statement

2 is correct).

2) It is the oldest extant text among the important works in Awadhi. A famous piece of Sufi

literature from the period, it relates an allegorical fictional story about the Delhi Sultan Alauddin

Khalji’s desire for the titular Padmavati, the Queen of Chittor. Alauddin Khalji and Padmavati’s

husband Ratan Sen are historical figures, whereas Padmavati is a fictional character.

Mathnawi or masnavi — a poem written in rhyming couplets, or more specifically, “a poem

based on independent, internally rhyming lines”. (Hence statement 1 is correct).

(Hence option c is correct)

Source: Tamil Nadu History Class 11 and Wiki

20. Which of the following is NOT correct about Hill temple architecture in Kashmir ?

a) It has a strong Gandhara influence.

b) They have influences of Hindu, Buddhist and Jain traditions.

c) Use of wood can be seen in these temples.

d) The Karkota Empire was the most significant in terms of architecture.

Insights IAS | InsightsonIndia

www.insightsias.com 21 www.insightsonindia.com

INS

IGH

TS

IAS

RE

VIS

ION

TE

ST

S F

OR

UP

SC

CIV

IL S

ER

VIC

ES

PR

EL

IMIN

AR

Y E

XA

M – 2

01

8

Solution: b

Justification:

Hill temple architecture in Kashmir

1) Place of development : hills of Kumaon, Garhwal, Himachal and Kashmir

2) Influence :

Kashmir’s proximity to prominent Gandhara sites (such as Taxila, Peshawar and the northwest

frontier) lent the region a strong Gandhara influence by the 5CE.

This began to mix with the Gupta and post-Gupta traditions that were brought to it from Sarnath,

Mathura and even centres in Gujarat and Bengal.

Brahmin pundits and Buddhist monks frequently travelled between Kashmir, Garhwal, Kumaon

and religious centres in the plains like Banaras, Nalanda and even as far south as Kanchipuram.

As a result both Buddhist and Hindu traditions began to intermingle and spread in the hills.

The hills also had their own tradition of wooden buildings with pitched roofs.

At several places in the hills, therefore, you will find that while the main garbhagriha and

shikhara are made in a rekha-prasada or latina style, the mandapa is of an older form of wooden

architecture. Sometimes, the temple itself takes on a pagoda shape.

The Karkota period of Kashmir is the most significant in terms of architecture. One of the most

important temples is Pandrethan, built during the eighth and ninth centuries.

(Hence option b is correct)

Source: NCERT class 11-An introduction to Indian Art part 1

21. Along with which other territory (ies) was Goa constituted as a union territory by the 12th

Constitution Amendment Act, 1962 ?

a) Dadar and Nagar Haveli

b) Daman and Diu

c) Puducherry

d) Chandigarh

Solution: b)

Goa, Daman and Diu India acquired these three territories from the Portuguese by means of a

police action in 1961. They were constituted as a union territory by the 12th Constitutional

Insights IAS | InsightsonIndia

www.insightsias.com 22 www.insightsonindia.com

INS

IGH

TS

IAS

RE

VIS

ION

TE

ST

S F

OR

UP

SC

CIV

IL S

ER

VIC

ES

PR

EL

IMIN

AR

Y E

XA

M – 2

01

8

Amendment Act, 1962. Later, in 1987, Goa was conferred a statehood. Consequently, Daman and

Diu was made a separate union territory.

22. Swayam Prabha’, a Government of India initiative, is a project for

a) Online training of Yoga and Meditation practitioners

b) Promoting entrepreneurship by way of the Government setting up state of the art skilling

centres in collaboration with the private industry

c) Identifying new digital technology innovations to solve the many social problems faced

by the poor and vulnerable in urban setups

d) Providing high quality educational content through DTH channels to teachers, students

and citizens across the country

Solution: d)

The Centre will formally launch Swayam Prabha – a project for telecasting ‘High-quality

educational programmes’ through 32 DTH channels on Sunday at a National Convention on

Digital Initiatives for Higher Education.

Every day, there will be content of four hours that will be repeated six times, thus covering all

24 hours of the day. This will allow the student to choose a time of his own convenience,” said

an official. “The channels will cover curriculum-based course content covering arts, science,

commerce, performing arts, social sciences, engineering, medicine, law, agriculture, etc.”

http://www.thehindu.com/education/schools/coming-direct-to-home-coaching-for-students-

iit-aspirants/article19240986.ece;

Improvisation: http://www.thehindu.com/news/national/more-channels-to-train-

teachers/article22339416.ece;

23. In what ways can extracting the genetic information of different Indian groups (genome

sequencing) be beneficial ?

1. The information can be utilised to help decrease the country’s disease burden

2. The application of this information will contribute to India becoming a knowledge

based-economy

Select the correct answer using the code given below:

a) 1 only

b) 2 only

c) Both 1 and 2

Insights IAS | InsightsonIndia

www.insightsias.com 23 www.insightsonindia.com

INS

IGH

TS

IAS

RE

VIS

ION

TE

ST

S F

OR

UP

SC

CIV

IL S

ER

VIC

ES

PR

EL

IMIN

AR

Y E

XA

M – 2

01

8

d) Neither 1 nor 2

Solution: c)

Knowledge is now recognised as the driver of productivity and economic growth, leading to a

new focus on the role of information, technology and learning in economic performance. The

term “knowledge-based economy” stems from this fuller recognition of the place of knowledge

and technology in modern economies.

To gain fully from the genomics revolution, India needs to collect information about the genetics

of its population and train manpower capable of interpreting it. The information that is needed

has to come from a large and sustained collection of data — fully sequenced individual genomes

along with medical histories for the individuals who volunteer for this effort. This knowledge

could then also be quickly applied to the task of managing diseases as well as be used for genetic

counselling that could reduce their incidence in future generations. As an example elsewhere,

the founder group of Ashkenazi Jews have almost eliminated Tay-Sachs disease from their

population by such means.

Q45, CS(P) 2017: http://www.insightsonindia.com/2017/06/19/answer-key-upsc-civil-

services-prelims-exam-2017-general-studies-paper-1/;

We are just providing the source for definition of KBE, no need to refer to it:

https://www.oecd.org/sti/sci-tech/1913021.pdf;

Source/improvisation: http://www.thehindu.com/opinion/lead/towards-a-genomics-

revolution/article22361004.ece;

24. Which among the following is NOT an official language of the United Nations ?

a) Arabic

b) Spanish

c) German

d) Russian

Solution: c)

There are six official languages of the UN. These are Arabic, Chinese, English, French, Russian

and Spanish. The correct interpretation and translation of these six languages, in both spoken

and written form, is very important to the work of the Organization, because this enables clear

and concise communication on issues of global importance.

Insights IAS | InsightsonIndia

www.insightsias.com 24 www.insightsonindia.com

INS

IGH

TS

IAS

RE

VIS

ION

TE

ST

S F

OR

UP

SC

CIV

IL S

ER

VIC

ES

PR

EL

IMIN

AR

Y E

XA

M – 2

01

8

A delegate may speak in any official UN language. The speech is interpreted simultaneously into

the other official languages of the UN.

At times, a delegate may choose to make a statement using a non-official language. In such cases,

the delegation must provide either an interpretation or a written text of the statement in one of

the official languages.

Most UN documents are issued in all six official languages, requiring translation from the original

document.

Additional Information:

Minister Sushma Swaraj said the process required a two-thirds majority vote, and that other

countries using Hindi should share the expenditure incurred in making the language an official

one.

Though there are six official languages in the U.N., only two — English and French — were

working languages.

http://www.un.org/en/sections/about-un/official-languages/;

Improvisation: http://www.thehindu.com/news/national/what-is-the-purpose-of-hindi-as-an-

official-language-in-un-asks-tharoor/article22357862.ece;

25. In contrast with the Medical Council of India, the proposed National Medical Council will

1. Centralise all powers from advising universities on curriculum to disciplining errant

doctors

2. Include among its members non-doctors like patient-rights advocates and ethicists

Select the correct answer using the code given below:

a) 1 only

b) 2 only

c) Both 1 and 2

d) Neither 1 nor 2

Solution: b.

In contrast with the MCI, which does everything from advising universities on curriculum to

disciplining errant doctors, the NMC distributes powers among four autonomous boards —

those for undergraduate education, postgraduate education, medical assessment and rating, and

ethics and registration. Also, unlike the MCI, the commission includes non-doctors like patient-

rights advocates and ethicists, in line with the medical regulators of the U.K., Australia and

Canada.

Insights IAS | InsightsonIndia

www.insightsias.com 25 www.insightsonindia.com

INS

IGH

TS

IAS

RE

VIS

ION

TE

ST

S F

OR

UP

SC

CIV

IL S

ER

VIC

ES

PR

EL

IMIN

AR

Y E

XA

M – 2

01

8

Where the NMC bill trips up is in how it chooses the members of the new regulator. The NMC

Bill’s solution to the pitfalls of the MCI electoral process is for the central government to select

most of the commission’s members… The Bill also misses an opportunity to plan for India’s rural

health-care needs (especially the issue of shortage of doctors) in the coming decades.

http://www.thehindu.com/opinion/op-ed/prescription-for-the-future/article22413556.ece;

http://www.prsindia.org/billtrack/the-national-medical-commission-bill-2017-5024/;

26. The farthest point from center of Earth is

a) Mount Kenya.

b) Mount Everest.

c) Mount Chimborazo.

d) Mauna Kea

Solution: c

Explanation: Highest above Earth’s Center: The Earth does not have the shape of a perfect

sphere. Instead, its diameter is greatest near the equator. In the diagram above, the gray dashed

line is a perfect circle, and the solid blue line represents the shape of the earth (exaggerated a bit

to make its departure from spherical obvious). Chimborazo is located near the equator where

Earth’s diameter is greatest. This makes the summit of Chimborazo the highest point above the

center of the Earth.

Mauna Kea: Tallest Mountain

Mauna Kea has an altitude of 4,205 meters (13,796 feet) – much lower than Mount Everest.

However, Mauna Kea is an island, and if the distance from the bottom of the nearby Pacific Ocean

floor to the peak of the island is measured, then Mauna Kea is “taller” than Mount Everest.

Insights IAS | InsightsonIndia

www.insightsias.com 26 www.insightsonindia.com

INS

IGH

TS

IAS

RE

VIS

ION

TE

ST

S F

OR

UP

SC

CIV

IL S

ER

VIC

ES

PR

EL

IMIN

AR

Y E

XA

M – 2

01

8

Mauna Kea is over 10,000 meters tall compared to 8,850 meters for Mount Everest – making it

the “world’s tallest mountain.”

27. Consider the following statements

1. The sun rays never fall vertically over the state of Manipur.

2. Eris earlier called Ceres 2003 UB313 is a dwarf planet located between Neptune and

Uranus.

Which of the following statement/s is/are correct ?

a) 1 Only

b) 2 Only

c) Both 1 and 2

d) None of the above

Solution: a

Explanation:

Manipur – The state lies at a latitude of 23°83’N – 25°68’N and a longitude of 93°03’E – 94°78’E.

Since it lies beyond Tropic of cancer line , the sun rays do not fall vertically on Manipur. (Hence

statement 1 is correct).

Eris is the largest known dwarf planet in the solar system. It is a trans-Neptunian object (TNO),

orbiting the Sun in a region of space known as the scattered disc, just beyond the Kuiper belt.

(Hence statement 2 is incorrect). Eris is the most distant object ever seen in orbit around the

Sun, even more distant than Sedna, the Kuiper Belt object discovered in 2003. It is almost 10

billion miles from the Sun and more than 3 times more distant than Pluto and takes more than

twice as long to orbit the Sun as Pluto. It has an orbital period of 556.7 years.

Insights IAS | InsightsonIndia

www.insightsias.com 27 www.insightsonindia.com

INS

IGH

TS

IAS

RE

VIS

ION

TE

ST

S F

OR

UP

SC

CIV

IL S

ER

VIC

ES

PR

EL

IMIN

AR

Y E

XA

M – 2

01

8

28. Consider the following statements

1. When crossing the International Date Line (IDL) into the Eastern Hemisphere, a person

loses a day.

2. The duration of continuous day or night decreases north ward as one moves from Artic

Circle to North Pole.

Which of the following statement/s is/are correct ?

a) 1 Only

b) 2 Only

c) Both 1 and 2

d) None of the above

Answer: a

The earth rotates from West to East, therefore, time to the East is ahead of time to the West.

When crossing the International Date Line from the Western to the Eastern Hemisphere we are

going ‘forward’ in terms of time and ‘backwards’ in time when crossing from the Eastern to the

Western Hemisphere. (Hence statement 1 is correct).

So, travel east across the International Date Line results in a day, or 24 hours, being subtracted.

Travel west across the International Date Line results in a day being added.

Day length varies with the time of year and the latitude at which you are situated. Because of the

earth’s tilt, areas above the Arctic Circle receive 24 hours of sunlight each summer, but 24 hours

of darkness each winter. The duration increases as we move from Arctic circle to North pole.

(Hence statement 2 is incorrect).

29. An important cause of the slowing down of the Earth’s rotation is the

a) Pull of the moon on the Earth’s Magnetic field

b) Tides caused by the gravity of the moon

c) Gravitational pull of sun

d) None of the above

Solution: b

Explanation:

The side of the Earth facing the Moon is about 6400 kilometers closer to the Moon than the center

of the Earth is, and the Moon’s gravity pulls on the near side of the Earth more strongly than on

the Earth’s center. This produces a tidal bulge on the side of the Earth facing the Moon. These

Insights IAS | InsightsonIndia

www.insightsias.com 28 www.insightsonindia.com

INS

IGH

TS

IAS

RE

VIS

ION

TE

ST

S F

OR

UP

SC

CIV

IL S

ER

VIC

ES

PR

EL

IMIN

AR

Y E

XA

M – 2

01

8

tidal bulges are always along the Earth-Moon line and the Earth rotates beneath the tidal bulge.

When the part of the Earth where you are located sweeps under the bulges, you experience a

high tide; when it passes under one of the depressions, you experience a low tide. An ideal coast

should experience the rise and fall of the tides twice a day. In reality, the tidal cycle also depends

on the latitude of the site, the shape of the shore, winds, etc. As the Earth rotates beneath the

tidal bulges, it attempts to drag the bulges along with it. A large amount of friction is produced

which slows down the Earth’s spin. The day has been getting longer and longer by about 0.0016

seconds each century.

(Hence answer is option B).

Rotational speed of earth is not affected by the magnetic lines

Because of the distance the impact of gravitational pull of sun on rotational speed variation is

negligible.

30. Identify the countries which border more than one ocean.

1. East Timor

2. Colombia

3. Thailand

Which of the following is/are correct ?

a) 1 and 2 Only

b) 1 and 3 only

c) 2 and 3 Only

d) All of the above

Answer: d

East Timor – Pacific and Indian Ocean

Colombia – Atlantic and Pacific

Thailand – Indian and Pacific oceans

Insights IAS | InsightsonIndia

www.insightsias.com 29 www.insightsonindia.com

INS

IGH

TS

IAS

RE

VIS

ION

TE

ST

S F

OR

UP

SC

CIV

IL S

ER

VIC

ES

PR

EL

IMIN

AR

Y E

XA

M – 2

01

8

31. Consider the following statements

1. Fold mountains are usually composed of sedimentary rocks

2. Block mountains are formed by the process of denudation

Which of the following statement/s is/are correct ?

a) 1 Only

b) 2 Only

c) Both 1 and 2

d) None of the above

Answer: a

Explanation:

Fold Mountains are formed when sedimentary rock strata in geosynclines are subjected to

compressive forces. Fold Mountains form when two tectonic plates move towards each other at

a convergent plate boundary. Fold Mountains form from sedimentary rocks that accumulate

along the margins of continents. (Hence statement 1 is correct).

Block Mountains are formed by faulting. Block mountains also known as Horst. It is formed when

the tensional forces causes the plates on the earth crusts to move apart. This movement causes

the rocks to be stretched. Temperatures are low and the rocks are brittle. Instead of folding, they

break into large blocks. (Hence statement 2 is incorrect).

Due to the compressional forces, the middle section of the rocks are uplifted, resulting it to be

higher than the surrounding land.

An example of the block mountain is the Vosges in France and the Black Forest in Germany which

are separated by the Rhine Valley.

Insights IAS | InsightsonIndia

www.insightsias.com 30 www.insightsonindia.com

INS

IGH

TS

IAS

RE

VIS

ION

TE

ST

S F

OR

UP

SC

CIV

IL S

ER

VIC

ES

PR

EL

IMIN

AR

Y E

XA

M – 2

01

8

32. Lakshadweep Island are the product of

a) Volcanic Activity

b) Wave action

c) Sea-floor expansion

d) Reef formation

Answer : d

Explanation : Lakshadweep islands consist of a group of tiny coral islands, located in the

Arabian Sea, about 400 km from the main land (southern tip of the Indian Peninsula). The Union

Territory of Lakshadweep consists of 10 inhabited islands, 17 uninhabited islands, attached

islets, 4 newly formed islets and 5 submerged reefs. These islands are scattered in the Arabian

Sea between North Latitudes 8o 00’ and 12o 13’N and east longitude 71o 00’ and 74o 00’E .

These islands are typically a chain of low islands surrounding a shallow lagoon, consisting

largely of recent sediments on top of older coral limestone.

33. Which of the following statements is NOT correct

a) Oceanic crust are much younger compared to Continental crust.

b) The oceanic crust is made of basalt and gabbro rocks

c) The Earth’s mantle is composed of ultramafic rocks

d) The largest portion of Earth’s volume is occupied by core.

Answer : d

Explanation :

Earth’s crust only accounts for less than 1% of the Earth’s interior. The Earth’s crust is

the outer shell of the Earth. This is the part that has cooled down enough to solidify into

rock. The crust extends down 30 km to 80 km underneath the continents, and only 5 km

beneath the oceans. As you travel down through the crust, temperatures increase. The

crust is broken up into several tectonic plates which “float” on top of the Earth’s mantle.

In some regions, plates are sliding underneath one another, recycling rocks into the Earth.

The crust beneath the middle of the oceans is spreading apart, and new material is welling

up.

Beneath the crust is the largest part of the Earth’s interior: the mantle, which makes up

about 84% of the Earth’s volume. This region extends down to a depth of 2,890 km. As

you travel down through the mantle, temperatures increase immensely;

Insights IAS | InsightsonIndia

www.insightsias.com 31 www.insightsonindia.com

INS

IGH

TS

IAS

RE

VIS

ION

TE

ST

S F

OR

UP

SC

CIV

IL S

ER

VIC

ES

PR

EL

IMIN

AR

Y E

XA

M – 2

01

8

At the very center of the Earth lies the core. This is a solid sphere of metal 2,440 km across

surrounded by a layer of liquid metal. Scientists think that mostly made of iron (80%),

with the rest composed of other heavy metals, like nickel, gold, platinum and even

uranium.

The oceanic crust are destroyed along the submergent boundaries ( trenches ) and new

oceanic floor is created along the divergent plate boundary

Mantle is composed of ultramafic rocks

Oceanic crust is composed of basalt

34. Which one of the following is the correct sequence of landforms developed under fluvial cycle

of erosion in terms of where they are formed in the river course ?

a) Gorge-Piedmont plain-Flood plain-Delta

b) Gorge-Flood plain-Piedmont plain-Delta

c) Piedmont Plain-Gorge-Flood Plain-Delta

d) Gorge-Piedmont Plain-Delta-Flood Plain

Answer : a

Explanation :

Piedmont plain are formed along the foothills region.

Gorge are formed at the upper reaches of mountains

Flood plains are formed at lower reaches where the elevation is much lower

Delta plains are formed along coastline

35. The factors that affect wind direction and speed are.

a) Pressure gradient force, coriolis force, moisture content.

b) Coriolis force, pressure gradient force, friction.

c) Friction, moisture content, pressure gradient force.

d) Moisture content, centripetal acceleration, coriolis force.

Answer: b

Insights IAS | InsightsonIndia

www.insightsias.com 32 www.insightsonindia.com

INS

IGH

TS

IAS

RE

VIS

ION

TE

ST

S F

OR

UP

SC

CIV

IL S

ER

VIC

ES

PR

EL

IMIN

AR

Y E

XA

M – 2

01

8

Explanation: Moisture does not affect the wind speed and direction

Factors Affecting Wind Motion:

1. Pressure Gradient Force:

This is the force generated due to the differences in horizontal pressure, and it operates from the

high pressure area to a low pressure area. Since a closely spaced gradient implies a steep

pressure change, it also indicates a strong wind speed. The wind direction follows the direction

of change of pressure, i.e. perpendicular to the isobars.

2. Coriolis Force:

Due to the earth’s rotation, winds do not cross the isobars at right angles as the pressure gradient

force directs, but get deflected from their original path. This deviation is the result of the earth’s

rotation and is called the Coriolis Effect or Coriolis force. Due to this effect, winds in the northern

hemisphere get deflected to the right of their path and those in the southern hemisphere to their

left, following Farrel’s Law. The Coriolis force changes wind direction but not its speed. This

deflection force does not seem to exist until the air is set in motion and increases with wind

velocity, air mass and an increase in latitude.

3. Centripetal Acceleration:

Due to inward acceleration of air towards the centre of rotation on the rotating earth, it is

possible for the air to maintain a curved path (parallel to the isobars), about a local axis of high

or low pressure. It is known as centripetal acceleration.

4. Frictional Force:

The irregularities of the earth’s surface offer resistance to the wind motion in the form of friction.

This force determines the angle at which air will flow across the isobars, as well as the speed at

which it will move. It may also alter wind direction. Over the relatively smooth ocean surface,

the friction is minimum, so the air moves at low angles to the isobars and at a greater speed. Over

uneven terrain, however, due to high friction, the wind direction makes high angles with, isobars

and the speed gets retarded.

Insights IAS | InsightsonIndia

www.insightsias.com 33 www.insightsonindia.com

INS

IGH

TS

IAS

RE

VIS

ION

TE

ST

S F

OR

UP

SC

CIV

IL S

ER

VIC

ES

PR

EL

IMIN

AR

Y E

XA

M – 2

01

8

36. Consider the following statements regarding ocean currents.

1. At higher latitudes, usually cold currents move along the western coasts of the continents.

2. Alaska current is a warm current moving along coast of USA and Canada.

Which of the following statement/s is/are correct ?

a) 1 Only

b) 2 Only

c) Both 1 and 2

d) None of the above

Answer : b

Explanation: In general the cold currents move eastern coast of continents at higher latitudes.

Example Labrador current , Oyashio current. (Hence statement 1 is incorrect).

Alaska is a warm current. (Hence statement 2 is correct).

Insights IAS | InsightsonIndia

www.insightsias.com 34 www.insightsonindia.com

INS

IGH

TS

IAS

RE

VIS

ION

TE

ST

S F

OR

UP

SC

CIV

IL S

ER

VIC

ES

PR

EL

IMIN

AR

Y E

XA

M – 2

01

8

37. Which of the following is NOT a feature of tropical rainforest.

a) More than 200cm of annual rainfall.

b) Nutrient-rich and pH-neutral soil

c) Low annual range of temperature.

d) Dense canopy with little light reaching the forest floor.

Answer : b

Explanation :

Most tropical rainforest soils relatively poor in nutrients. Millions of years of weathering and

torrential rains have washed most of the nutrients out of the soil. Constant warmth and moisture

promote rapid decay of organic matter. When a tree dies in the rainforest, living organisms

quickly absorb the nutrients before they have a chance to be washed away. When tropical forests

are cut and burned, heavy rains can quickly wash the released nutrients away, leaving the soil

even more impoverished. Because of heavy leaching of bases the soil is acidic in nature

38. Between which one of the following pairs of Ports does Panama Canal shorten the distance

to the maximum.

a) Liverpool and shanghai

b) New York and wellington

c) Liverpool and Sydney

d) New York and San Francisco.

Answer : d

Explanation:

Because of the circumnavigation of South America the distance is shortened to maximum

between New York and San Francisco. In rest of the cases the proportion of distance shortened

is lesser.

Insights IAS | InsightsonIndia

www.insightsias.com 35 www.insightsonindia.com

INS

IGH

TS

IAS

RE

VIS

ION

TE

ST

S F

OR

UP

SC

CIV

IL S

ER

VIC

ES

PR

EL

IMIN

AR

Y E

XA

M – 2

01

8

39. Consider the following statements regarding HOT deserts.

1. High annual range of temperature.

2. Nomadism

3. Located on eastern margins of continent.

Which of the following are CORRECT ?

a) 2 Only

b) 1 & 2 Only

c) 2 & 3 Only

d) 1 & 3 only

Answer: a

Explanation: The annual range of temperature is lesser and the temperatures are higher

throughout the year. Hot deserts have greater diurnal range of temperature. (Hence statement

1 is incorrect).

Hot deserts are located on the Western margins of continents. (Hence statement 3 is incorrect).

Nomadism is important characteristics of deserts where people or tribe move place to place in

search of pasture and food. (Hence statement 2 is correct).

40. Which of the following are correctly matched.

Forest Tree

1. Tundra Vegetation — Lichens

2. Tropical Deciduous Forest — Mahogany

3. Coniferous Forests — Pine

a) 3 Only

b) 1 Only

c) 1 & 3 Only

d) 1 & 2 Only

Answer: c

Explanation:

Mahogany is found in the Tropical Evergreen forest.

Insights IAS | InsightsonIndia

www.insightsias.com 36 www.insightsonindia.com

INS

IGH

TS

IAS

RE

VIS

ION

TE

ST

S F

OR

UP

SC

CIV

IL S

ER

VIC

ES

PR

EL

IMIN

AR

Y E

XA

M – 2

01

8

Tundra Vegetation is found in polar areas of Europe, Asia and North America. Only mosses ,

lichens and very small shrubs are found in these regions

Coniferous forests are mostly found in regions of the Earth that experience long winters and

short summers. Coniferous forests, therefore, are found mainly in the northern hemisphere,

although some are found in the southern hemisphere.Coniferous forests (fir, pine, spruce) make

up one-third of the world’s forests and are found in northern parts of North America, Europe

and Asia where temperatures tend to be lower and winter tends to last longer.

41. Which of the following rivers originate along the slopes of Eastern Ghats

1. Vamshadhara

2. Subranarekha

3. Indravati

4. Vaigai

Which of the following is CORRECT ?

a) 1 and 4

b) 1 and 3

c) 1, 2 and 3

d) 2, 3 and 4

Answer: c

Explanation :

Vaigai originates from western ghats and falls in Palk starit.

Subarnarekha rivers originates in chotanagpur plateau near Ranchi and flows through the states

of Jharkand, West Bengal and Odisha.

Vamshadhara and Indravati rivers originate along slopes of Eastern Ghats.

42. Which of the following rivers form natural boundary between various states in India.

1. Ravi

2. Chambal

3. Yamuna

4. Seonath

a) 1, 2, 3 b) 1, 2 & 4

c) 2, 3 Only d) 2, 3 &4

Insights IAS | InsightsonIndia

www.insightsias.com 37 www.insightsonindia.com

INS

IGH

TS

IAS

RE

VIS

ION

TE

ST

S F

OR

UP

SC

CIV

IL S

ER

VIC

ES

PR

EL

IMIN

AR

Y E

XA

M – 2

01

8

Answer : a

Explanation:

Ravi river forms boundary between Jammu Kashmir and Punjab.

Chambal river forms boundary between Rajasthan and Madhya Pradesh.

Yamuna river forms boundary between Haryana and Uttarpradesh.

Seonath or shivanath is tributary of Mahanadi river entirely located in Chattisgarh

43. Which of the following Ramsar Sites are located in Western Himalayas.

1. Nalasarovar

2. Kanjli

3. Rudrasagar Lake

a) 2 & 3 only

b) 3 Only

c) 1 & 3 Only

d) None of the above

Answer : d

Nalasarovar is in Gujarat

Kanjli is in Punjab plains

Rudrasagar lake is in Tripura

44. Identify the brackish/salt water lakes India.

1. Pangong Tso

2. Sambhar Lake

3. Loktak Lake

a) 1 & 3

b) 2 Only

c) 1 & 2

d) 2 & 3

Answer : c

Insights IAS | InsightsonIndia

www.insightsias.com 38 www.insightsonindia.com

INS

IGH

TS

IAS

RE

VIS

ION

TE

ST

S F

OR

UP

SC

CIV

IL S

ER

VIC

ES

PR

EL

IMIN

AR

Y E

XA

M – 2

01

8

Expalnation :

Pangong Tso formed from Tethys geosyncline is a salt water lake.

Sambhar is salt water lake in Rajasthan where evaporation exceeds the precipitation.

Loktak lake is fresh water lake in Manipur

45. Doyang and Dikhu are the prominent river systems of which of the following state.

a) Arunachal Pradesh

b) Mizoram

c) Meghalaya

d) Nagaland

Answer : d

Explanation: The major rivers of Nagaland are Doyang, Dikhu, Dhansiri, Tizu, Tsurong, Nanung,

Tsurang or Disai, Tsumok, Menung, Dzu, Langlong, Zunki, Likimro, Lanye, Dzuza and Manglu. All

these rivers are dendritic in nature.

Of the rivers, Dhansiri, Doyang and Dikhu flow westward into the Brahmaputra.

The Tizu River, on the other hand, flows towards east and joins the Chindwin River in Burma.

46. The General Data Protection Regulation (GDPR) is a regulation that intends to strengthen

and harmonise data protection law frameworks across the

a) European Union

b) G20 group of nations

c) OECD countries

d) NATO countries

Solution: a.

GDPR seeks to create a harmonised data protection law framework across the EU and aims to

give citizens back the control of their personal data, whilst imposing strict rules on those hosting

and ‘processing’ this data, anywhere in the world. The Regulation also introduces rules relating

to the free movement of personal data within and outside the EU.

Insights IAS | InsightsonIndia

www.insightsias.com 39 www.insightsonindia.com

INS

IGH

TS

IAS

RE

VIS

ION

TE

ST

S F

OR

UP

SC

CIV

IL S

ER

VIC

ES

PR

EL

IMIN

AR

Y E

XA

M – 2

01

8

GDPR would apply to all firms, regardless of their location, that process personal data of people

living in the EU. Once in force, the GDPR would require companies to notify a data breach within

72 hours of the event.

What is the GDPR: https://www.ibm.com/analytics/us/en/technology/general-data-

protection-regulation/; https://www.eugdpr.org;

Source/Improvisation: http://www.thehindu.com/business/Economy/too-few-prepared-for-

gdpr/article22612475.ece;

47. Consider the following statements with reference to inflation trends in the Indian economy

over the past few years:

1. The average Consumer Price Index has steadily declined since 2014-15

2. The average Wholesale Price Index declined in 2015-16 but has been rising since

Which of the statements given above is/are correct ?

a) 1 only

b) 2 only

c) Both 1 and 2

d) Neither 1 nor 2

Solution: c)

http://pib.nic.in/newsite/PrintRelease.aspx?relid=175979

http://www.insightsonindia.com/2018/02/01/quiz-2017-insights-current-affairs-quiz-01-

february-2018/

48. The Technical Education Quality Improvement Programme (TEQIP) is run by

a) Ministry of Labour and Employment

b) Ministry of Human Resource Development

c) Ministry of Micro, Small and Medium Enterprises

d) Ministry of Skill Development and Entrepreneurship

Solution: b.

Insights IAS | InsightsonIndia

www.insightsias.com 40 www.insightsonindia.com

INS

IGH

TS

IAS

RE

VIS

ION

TE

ST

S F

OR

UP

SC

CIV

IL S

ER

VIC

ES

PR

EL

IMIN

AR

Y E

XA

M – 2

01

8

The Ministry of Human Resource Development has recruited more than 1200 highly qualified

and motivated graduates from IITs and NITs among others to teach in Engineering Colleges in

the most backward areas for the next 3 years.

Under this project (presently TEQIP-III, assisted by World Bank)), all Government engineering

colleges are selected for direct intervention, and all private engineering colleges are selected for

indirect intervention. The measures are institution-based (ex: governance reforms) and student-

based (ex: improving the quality of teaching).

http://pib.nic.in/PressReleseDetail.aspx?PRID=1518453

http://mhrd.gov.in/technical-education-12

49. Consider the following statements:

1. It is a low cost, solar energy driven cleanup technique.

2. It is most useful at sites with shallow, low levels of contamination.

3. It is Useful for treating a wide variety of environmental contaminants.

4. It is effective with, or in some cases, in place of mechanical cleanup methods

The above statements refer to:

a) Phytoremediation

b) Bioventing

c) Bioleaching

d) Bioremediation

Solution: a)

http://www.insightsonindia.com/2018/02/03/quiz-2017-insights-current-affairs-quiz-03-

february-2018/

Phytoremediation (‘phyto’ means plant) is a generic term for the group of technologies that use

plants for remediating soils, sludges, sediments and water contaminated with organic and

inorganic contaminants. Phytoremediation can be defined as “the efficient use of plants to

remove, detoxify or immobilise environmental contaminants in a growth matrix (soil, water or

sediments) through the natural biological, chemical or physical activities and processes of the

plants”. Plants are unique organisms equipped with remarkable metabolic and absorption

capabilities, as well as transport systems that can take up nutrients or contaminants selectively

from the growth matrix, soil or water. Phytoremediation involves growing plants in a

contaminated matrix, for a required growth period, to remove contaminants from the matrix, or

facilitate immobilisation (binding/containment) or degradation (detoxification) of the

pollutants. The plants can be subsequently harvested, processed and disposed.

http://www.unep.or.jp/Ietc/Publications/Freshwater/FMS2/1.asp

Insights IAS | InsightsonIndia

www.insightsias.com 41 www.insightsonindia.com

INS

IGH

TS

IAS

RE

VIS

ION

TE

ST

S F

OR

UP

SC

CIV

IL S

ER

VIC

ES

PR

EL

IMIN

AR

Y E

XA

M – 2

01

8

50. Consider the following statements:

1. The Parliament can amend a repealed legislation retrospectively

2. The finance bill, the passing of which by the Parliament is a part of enactment of the

budget, is a money bill

Which of the statements given above is/are correct ?

a) 1 only

b) 2 only

c) Both 1 and 2

d) Neither 1 nor 2

Solution: c.

Presently, consider statement 1 to be correct. Keep track of the news for any updates on this

issue.

The Union government has, in the Budget, proposed to amend the repealed Foreign Contribution

Regulation Act (FCRA), 1976, retrospectively, a move that will benefit the ruling BJP and the

Congress held guilty by the Delhi High Court for receiving foreign funds from two subsidiaries of

Vedanta, a U.K.-based company. (The Representation of the People Act and the FCRA bar

political parties from receiving foreign funds.)

In 2016, the government amended the FCRA through the Finance Bill route, allowing foreign-

origin companies to finance non-governmental organisations and clearing the way for donations

to political parties by changing the definition of “foreign companies”. The amendment, though

done retrospectively, only made valid the foreign donations received after 2010, the year when

the 1976 Act was amended. The retrospective amendment did not apply to donations prior to

2010.

Justification for Statement 2 (Stages in Enactment of Budget -> Chapter ‘Parliament’, Indian

Polity by M Laxmikanth): The Finance Bill (not financial bill) is introduced to give effect to the

financial proposals of the Government of India for the following year. It is subjected to all

the conditions applicable to a Money Bill.

http://www.thehindu.com/todays-paper/tp-national/fcra-changes-will-help-bjp-

cong/article22639424.ece;

http://indianexpress.com/article/cities/delhi/fcra-amendment-in-finance-bill-to-help-bjp-

congress-escape-scrutiny-5049589/

Insights IAS | InsightsonIndia

www.insightsias.com 42 www.insightsonindia.com

INS

IGH

TS

IAS

RE

VIS

ION

TE

ST

S F

OR

UP

SC

CIV

IL S

ER

VIC

ES

PR

EL

IMIN

AR

Y E

XA

M – 2

01

8

51. The most suitable method for prevention of gully erosion is:

a) Mulching

b) Rock dam structure

c) Ploughing

d) Perimeter runoff control

Answer: (b)

Justification:

Insights IAS | InsightsonIndia

www.insightsias.com 43 www.insightsonindia.com

INS

IGH

TS

IAS

RE

VIS

ION

TE

ST

S F

OR

UP

SC

CIV

IL S

ER

VIC

ES

PR

EL

IMIN

AR

Y E

XA

M – 2

01

8

52. The relative proportion of sand, silt and clay in a soil refers to its

a) Profile

b) Horizon

c) Structure

d) Texture

Answer: (d)

Justification:

A soil profile is a vertical cross section of the soil. When exposed, various soil horizons, or layers

of soil, become apparent. Each horizon of soil may be different from the other horizons in

physical or chemical ways. The differences are developed from the interaction of such soil-

forming factors as parent material, slope, native vegetation, weathering, and climate.

Texture indicates the relative content of particles of various sizes, such as sand, silt and clay in

the soil. Texture influences the ease with which soil can be worked, the amount of water and air

it holds, and the rate at which water can enter and move through soil.

Soil structure is defined by the way individual particles of sand, silt, and clay are assembled.

Single particles when assembled appear as larger particles. These are called aggregates. Soil

structure is most usefully described in terms of grade (degree of aggregation), class (average

size) and type of aggregates (form).

53. Which of the following are correctly matched ?

Minerals Region

1. Coal — Appalachian

2. Tin — Andes region

3. Iron-ore — Urals

Which of the following is/are correct

a) 1 Only

b) 1 and 3 only

c) and 3 only

d) 1, 2 and 3

Answer: (d)

Insights IAS | InsightsonIndia

www.insightsias.com 44 www.insightsonindia.com

INS

IGH

TS

IAS

RE

VIS

ION

TE

ST

S F

OR

UP

SC

CIV

IL S

ER

VIC

ES

PR

EL

IMIN

AR

Y E

XA

M – 2

01

8

Justification:

Appalachian Mountains is rich in coal deposits. The Appalachian Province contains two-fifths

of bituminous coal and nearly all the anthracites.

Bolivia is one the leading producer of Tin in the world. Bolivian Andes are famous for tin

deposits.

Ural has rich deposits of iron ore. Magnitogorsk is an important mining area in Urals.

54. Consider the following statements

1. Ocean Thermal Energy is due to pressure difference at different levels in the ocean.

2. Ladakh a cold desert in Jammu Kashmir has huge solar energy potential.

Which of the following statement/s is/are correct ?

a) 1 Only

b) 2 Only

c) Both 1 and 2

d) All of the above

Answer: (b)

Justification:

Ocean Thermal Energy Conversion (OTEC) is a process that can produce electricity by using

the temperature difference between deep cold ocean water and warm tropical surface waters.

OTEC plants pump large quantities of deep cold seawater and surface seawater to run a power

cycle and produce electricity.

The Ladakh region of Jammu & Kashmir also known as cold desert has a vast potential to

produce 111 giga watts (GWS) of solar power.

The above diagram shows amount of insolation received by different regions.

Barren land, clear sky and greater insolation makes it an ideal location for solar energy projects.

Insights IAS | InsightsonIndia

www.insightsias.com 45 www.insightsonindia.com

INS

IGH

TS

IAS

RE

VIS

ION

TE

ST

S F

OR

UP

SC

CIV

IL S

ER

VIC

ES

PR

EL

IMIN

AR

Y E

XA

M – 2

01

8

Insights IAS | InsightsonIndia

www.insightsias.com 46 www.insightsonindia.com

INS

IGH

TS

IAS

RE

VIS

ION

TE

ST

S F

OR

UP

SC

CIV

IL S

ER

VIC

ES

PR

EL

IMIN

AR

Y E

XA

M – 2

01

8

55. Which one of the following statements is NOT CORRECT ?

a) In intensive subsistence agriculture, animal farming is more developed.

b) Shifting cultivation is characterized by field rotation rather than by crop rotation

c) Plantation agriculture is both capital and labor intensive

d) Nomadic herding is a form of pastoralism which arises in response to climatic

constraints and terrain

Answer: (a)

Justification:

Because of immense population pressure, most of land is utilized for cultivation of more than

one set of crops. Cattle rearing is not well developed because of very low availability of pasture

land. Cattle rearing is well developed in temperate grasslands. Intensive subsistence agriculture

is important feature of south and Southeast Asia.

Shifting agriculture, system of cultivation that preserves soil fertility by plot (field) rotation, as

distinct from crop rotation. In shifting agriculture a plot of land is cleared and cultivated for a

short period of time; then it is abandoned and allowed to revert to its natural vegetation while

the cultivator moves on to another plot. The period of cultivation is usually terminated when the

soil shows signs of exhaustion or, more commonly, when the field is overrun by weeds

Plantation agriculture is a form of commercial farming where crops are grown for profit. The

characteristic features of this type of farming are large estates or plantations, large capital

investment, managerial and technical support, scientific methods of cultivation, single crop

specialization, cheap labor, and a good system of transportation which links the estates to the

factories and markets for the export of the products

Nomadism is practiced in semi and arid regions of tropical regions and tundra. Because of

climatic constraints tribes wander in search of food and pasture.

56. Which of the following are correctly matched ?

1. Nomadic Herding – Tundra Region.

2. Mixed Farming – Eastern USA

3. Commercial Grain Farming – South East Asia.

Insights IAS | InsightsonIndia

www.insightsias.com 47 www.insightsonindia.com

INS

IGH

TS

IAS

RE

VIS

ION

TE

ST

S F

OR

UP

SC

CIV

IL S

ER

VIC

ES

PR

EL

IMIN

AR

Y E

XA

M – 2

01

8

Which of the following is/are correct.

a) 2 Only

b) 2 and 3 only

c) 1 and 2 only

d) All the above

Answer: (c)

Justification:

Nomadic Herders of Tundra

On the southern margins of Tundra, there are certain nomadic herd­ers, e.g., Lapps, Yakuts and

Eskimos. These herders have adjusted themselves to the food supply of reindeer. In the northern

parts of Norway, Sweden, Russia and Finland such herders constitute a sig­nificant part of the

population.

During the short summer they live on the grassy mountains and in autumn they migrate along

with their herds in coniferous areas of the south in order to meet the fodder re­quirements of

their herds and to protect them from the freezing conditions of the Tundra climate.

Eastern USA is industrialized and densely populated region. In mixed farming both agriculture

cultivation and cattle rearing is well developed. Mixed farming is important characteristic of

well-developed region with greater population, for e.g. Western Europe.

Commercial grain farming is important feature of temperate grasslands.

57. Consider the following crops cultivated in India.

1. Cardamom

2. Tea

3. Coffee

Which of the above in general are grown under shade of trees or partial sunlight ?

a) 3 Only

b) 1 and 3 Only

c) 2 and 3 Only

d) All the above

Answer: (b)

Insights IAS | InsightsonIndia

www.insightsias.com 48 www.insightsonindia.com

INS

IGH

TS

IAS

RE

VIS

ION

TE

ST

S F

OR

UP

SC

CIV

IL S

ER

VIC

ES

PR

EL

IMIN

AR

Y E

XA

M – 2

01

8

Justification:

Cardamom is grown commercially in plantations under the shade of tall forest trees. It is a very

labor-intensive crop to produce. The fruits are picked individually by hand before they are fully

ripe, over a period of several months. In India, cardamom grows under natural conditions of the

evergreen forests in the Western Ghats. It thrives best in tropical forests at altitudes ranging

from 600-1500 metres, receiving a well-distributed rainfall of over 150 cm and a temperature

of 10o – 35o C. Its optimum growth and development is observed in warm and humid places

under the canopy of lofty, evergreen forest trees. It is highly sensitive to wind and drought and,

therefore areas liable to be affected by these conditions are unsuitable. The crop does not survive

in waterlogged or excessive moisture.

Coffee plant requires hot and humid climate with temperature varying between 15°C and 28 °C

and rainfall from 150 to 250 cm. It does not tolerate frost, snowfall, high temperature above 30°C

and strong sun shine and is generally grown under shady trees. Prolonged drought is also

injurious to coffee. Dry weather is necessary at the time of ripening of the berries. Stagnant water

is harmful and this crop is grown on hill slopes at elevations from 600 to 1,600 metres above sea

level.

Tea

Temperature: 21°C to 29°C is ideal for the production of tea. High temperature is

required in summer. The lowest temperature for the growth of tea is 16°C.

Rainfall: 150-250 cm of rainfall is required for tea cultivation.

Soil: Tea shrubs require fertile mountain soil mixed with lime and iron. The soil should

be rich in humus.

Land: Tea cultivation needs well drained land. Stagnation of water is not good for tea

plants. Heavy rainfall but no stagnancy of water, such mountain slopes are good for tea

cultivation.

58. Consider the following statements

1. Intensive subsistence agriculture is important characteristic of fertile plains of South and

South-East Asia with higher density of population.

2. In general per hectare output in commercial agriculture is much higher compared to

Intensive agriculture.

Which of the following statement/s is/are correct ?

a) 1 Only b) 2 Only

c) Both 1 and 2 d) None of the above

Insights IAS | InsightsonIndia

www.insightsias.com 49 www.insightsonindia.com

INS

IGH

TS

IAS

RE

VIS

ION

TE

ST

S F

OR

UP

SC

CIV

IL S

ER

VIC

ES

PR

EL

IMIN

AR

Y E

XA

M – 2

01

8

Answer: (b)

Justification:

Fertile alluvial plains of south and south-east Asia practice intensive subsistence

agriculture. For e.g Indus, Gangetic plains.

The per hectare productivity of intensive agriculture is much higher. Fertile soil, greater

labor and agricultural inputs leads to greater productivity.

59. According to most scientists, which of the following is NOT true about the clearing of Tropical

rainforests.

a) It enlarges holes in the ozone layer

b) It causes global warming

c) It negatively impacts the quality of soil.

d) It is partly caused by shifting cultivation.

Answer: (a)

Justification:

Why does the ozone hole form over Antarctica?

The answer is essentially ‘because of the weather in the ozone layer’. In order for rapid ozone

destruction to happen, clouds (known as PSCs, Stratospheric Clouds Mother of Pearl or

Nacreous Clouds) have to form in the ozone layer. In these clouds surface chemistry takes place.

This converts chlorine or bromine (from CFCs and other ozone depleting chemicals) into an

active form, so that when there is sunlight, ozone is rapidly destroyed. Without the clouds, there

is little or no ozone destruction. Only during the Antarctic winter does the atmosphere get cold

enough for these clouds to form widely through the centre of the ozone layer. Elsewhere the

atmosphere is just too warm and no clouds form. The northern and southern hemispheres have

different ‘weather’ in the ozone layer, and the net result is that the temperature of the Arctic

ozone layer during winter is normally some ten degrees warmer than that of the Antarctic. This

means that such clouds are rare, but sometimes the ‘weather’ is colder than normal and they do

form. Under these circumstances significant ozone depletion can take place over the Arctic, but

it is usually for a much shorter period of time and covers a smaller area than in the Antarctic.

Clearing of tropical forests leads to global warming, soil erosion and nutrient depletion.

Insights IAS | InsightsonIndia

www.insightsias.com 50 www.insightsonindia.com

INS

IGH

TS

IAS

RE

VIS

ION

TE

ST

S F

OR

UP

SC

CIV

IL S

ER

VIC

ES

PR

EL

IMIN

AR

Y E

XA

M – 2

01

8

60. Consider the following Tropical crops, which are in great demand in Anglo-America and

Europe, are produced in great variety and volume in Latin America and Africa because of

1. Availability of a cultivable land and suitability of soil conditions.

2. Combination of climatic and topographical conditions

3. Lack of mineral resources for trading purpose.

Which of the following is /are correct ?

a) 1 Only

b) 2 and 3 Only

c) 1 and 2

d) All the above

Answer: (c)

Justification:

Factors influencing the cultivation of crops are topography, climatic conditions and soil.

Because of suitable factors Africa and Latin America are important producers of plantation and

cash crops which is in great in Europe and America. Latin America and Africa are rich in mineral

resource which does not determine cultivation of tropical crops.

61. A triangular population pyramid with a wide base narrowing as the age cohort progress

indicates:

a) Decline

b) Rapid Growth

c) Slow Growth

d) Stability

Answer: (b)

Justification:

Rapid growth pyramids • Have a large base to show high birth rates • Amount of people

decreases as the ages goes up indicating a lower standard of living • Associated with developing

countries like Brazil, Uganda, China.

Insights IAS | InsightsonIndia

www.insightsias.com 51 www.insightsonindia.com

INS

IGH

TS

IAS

RE

VIS

ION

TE

ST

S F

OR

UP

SC

CIV

IL S

ER

VIC

ES

PR

EL

IMIN

AR

Y E

XA

M – 2

01

8

Slow growth pyramids • Take on a more rectangular shape • Indicates population is remaining

fairly steady • Birth rates and death rates are similar • Associated with developed countries like

the UK, Germany, Canada.

Negative growth pyramids • Looks like a reverse pyramid • Indicates the population of the

country is decreasing • Death rates are higher than birth rates • Associated with developed

countries like Austria, Japan, Italy.

Stable population: A pyramids with proportional sides – almost equal base, mid-section and

top. It is usually bell shaped pyramid. Countries with stable populations include France,

Scandinavian countries and Canada.

62. A country with a natural increase rate at or about zero is likely to have a

a) Low fertility rate, low standard of living and few people employed in agriculture.

b) Low fertility rate, low standard of living and many people employed in agriculture

c) Low fertility rate, high standard of living and few people employed in agriculture.

d) Low fertility rate, high standard of living and many people employed in agriculture.

Answer: (c)

Justification:

A lower fertility rate indicates a high standard of living. A high fertility rate indicates a low

standard of living. . A country with a high standard of living will have access to technology, and

therefore, will have relatively few people employed in agriculture,

63. Which of the following industries will most likely locate closest to its market.

a) Automobile assembly

b) Ethanol production

c) Telemarketing

d) Textile manufacturing.

Answer: (a)

Insights IAS | InsightsonIndia

www.insightsias.com 52 www.insightsonindia.com

INS

IGH

TS

IAS

RE

VIS

ION

TE

ST

S F

OR

UP

SC

CIV

IL S

ER

VIC

ES

PR

EL

IMIN

AR

Y E

XA

M – 2

01

8

Justification:

Automobile manufacturing is a weight-gaining industry. Hence it is located near to market.

Ethanol production is weight-losing industry, so it would be located close to their raw

materials.

Telemarketing is a footloose industry and can locate anywhere.

Textile manufacturing is a raw-material and labor-intensive industry and will usually locate

wherever labor is least expensive.

64. A country’s literacy rate has a positive correlation with all of the following EXCEPT

a) Energy Consumption

b) Per capita Income

c) Percentage of female employment

d) Birth rate.

Answer: (d)

Justification:

A country with higher literacy usually has low birth rate. Greater literacy rate is characteristic of

well developed economies where per capita income, energy consumption and female

employment is higher.

65. Consider the following

1. Aravallis

2. Vindhayas

3. Thar Desert

4. Satpuras

Which of the following extend over the state of Gujarat ?

a) 1 and 4 Only

b) 1, 2 and 4 only

c) 2, 3 and 4 only

d) All of the above

Insights IAS | InsightsonIndia

www.insightsias.com 53 www.insightsonindia.com

INS

IGH

TS

IAS

RE

VIS

ION

TE

ST

S F

OR

UP

SC

CIV

IL S

ER

VIC

ES

PR

EL

IMIN

AR

Y E

XA

M – 2

01

8

Answer: (d)

Justification:

All the four physiographic units extend into Gujarat. Rajpipla hills is part of Satpura range

extending into Gujarat.

66. Khardung La known for one of world’s highest road pass is located in which range.

a) Zanskar range

b) Ladakh range

c) Pirpanjal range

d) Baltoro Kangri

Answer: (b)

Justification:

Khardung La is a mountain pass in the Ladakh region of the Indian state of Jammu and Kashmir.

The pass on the Ladakh Range lies north of Leh and is the gateway to the Shyok and Nubra

valleys. The mountain pass holds a special significance as it is the way to carry supplies to the

Siachen Glacier.

67. Consider the following statements

1. Both Eastern Ghats and Western Ghats extend in the state.

2. Northern part of state is composed of Black soil.

3. The state has the smallest coastline in southern India

Which of the following states has the above features ?

a) Tamil Nadu

b) Andhra Pradesh

c) Kerala

d) Karnataka

Insights IAS | InsightsonIndia

www.insightsias.com 54 www.insightsonindia.com

INS

IGH

TS

IAS

RE

VIS

ION

TE

ST

S F

OR

UP

SC

CIV

IL S

ER

VIC

ES

PR

EL

IMIN

AR

Y E

XA

M – 2

01

8

Answer: (d)

Justification:

Both Western and Eastern Ghats extend in Karnataka. Biligirirangana hills are connecting link

between Western and Eastern Ghats.

Northern part of state is part of Deccan plateau .Northern part is predominantly black soil region

With around 320 km coastline it is smallest in South India.

68. Consider the following statements

1. Portions of Thar desert extends into Punjab

2. Shiwaliks extends over the northern part of Haryana.

Which of the above statement/s is/are correct.

a) 1 Only

b) 2 Only

c) Both 1 and 2

d) None of the above.

Answer: (c)

Justification:

Thar Desert, also called Great Indian Desert, arid region of rolling sand hills on the Indian

subcontinent. It is located partly in Rajasthan state, northwestern India, and partly in Punjab and

Sindh (Sind) provinces, eastern Pakistan.

Haryana has the Shivalik hills in the north and a range of Aravalli hills, in South West which runs

through southern Delhi and the Gurgaon district up to Alwar. These hills are known as the Morni

and Tipra ranges. They belong to the outer ranges of the Himalayas.

69. Which of the following pairs of hill station and range/hills is/are correctly matched?

Hill station Range /hills

1. Matheran — Eastern Ghats

2. Saputara — Western Ghats.

3. Pachmarhi — Vindhayas

Insights IAS | InsightsonIndia

www.insightsias.com 55 www.insightsonindia.com

INS

IGH

TS

IAS

RE

VIS

ION

TE

ST

S F

OR

UP

SC

CIV

IL S

ER

VIC

ES

PR

EL

IMIN

AR

Y E

XA

M – 2

01

8

Select the correct answer using the codes below.

a) 2 and 3 Only

b) 2 Only

c) 1 and 3 Only

d) 1 and 2 Only

Answer: (b)

Justification :

Matheran is a hill station and a municipal council in Karjat Tahsilin the Raigad district in the

Indian state of Maharashtra. The smallest hill station in India, it is located on the Western Ghats

range at an elevation of around 800 m (2,625 feet) above sea level.

Saputara is situated in the Dang district of Gujarat. This beautiful hill station is situated at an

elevation of about 1000 m above sea level in western ghats.

Pachmarhi is a hill station in Hoshangabad district of Madhya Pradesh state of central India. It

has been the location of a cantonment (Pachmarhi Cantonment) since British Raj. It is widely

known as Satpura ki Rani (“Queen of Satpura”), situated at a height of 1067 m in a valley of the

Satpura Range in Hoshangabad district.

70. Saddle Peak, the highest peak of Andaman and Nicobar islands is located in

a) South Andamans

b) Little Andamans

c) North Andamans

d) Car Nicobar

Answer: (c)

71. Consider the following statements

1. Karaikal, a district of Puducherry is an enclave of Kerala.

2. Telangana does not share boundary with Odisha.

Which of the following statements is/are correct.

a) 1 Only b) 2 Only

c) Both 1 and 2 d) None of the above

Insights IAS | InsightsonIndia

www.insightsias.com 56 www.insightsonindia.com

INS

IGH

TS

IAS

RE

VIS

ION

TE

ST

S F

OR

UP

SC

CIV

IL S

ER

VIC

ES

PR

EL

IMIN

AR

Y E

XA

M – 2

01

8

Answer: (b)

Justification:

Karaikal district, part of the discontiguous union territory of Pondicherry, is a small enclave

within Tamil Nadu. Karaikal and the other parts of Pondicherry are former French colonial

territories in India.

Telangana is bordered by the states of Maharashtra to the north, Chhattisgarh to the northeast,

Andhra Pradesh to the southeast and south, and Karnataka to the west.

72. Consider the following statements

1. The Peninsular India was part of Gondwanaland

2. The Gondwana consists of South America, Madgascar, Australia and India.

3. Peninsular plateau is the oldest and stable landmass composed of old crystalline igneous

and metamorphic rocks.

Which of the following is/are correct.

a) 1 & 3 Only

b) 2 & 3 Only

c) 1 & 2 Only

d) All the above.

Answer: (d)

Justification:

Gondwana (originally Gondwanaland) is an hypothesized ancient southern supercontinent

comprising most of the landmasses in today’s southern hemisphere, including Antarctica, South

America, Africa, Madagascar, Australia-New Guinea and New Zealand, as well as Arabia and the

Indian subcontinent, which are in the Northern Hemisphere.

The peninsular plateau is a table land composed of the old crystalline, igneous and metamorphic

rocks. It was formed due to breaking and drifting of the Gondwana land and thus making it a part

of oldest landmass. This plateau has broad and shallow valleys and rounded hills. The peninsular

plateau consists of broad divisions, namely, the central highlands and the Deccan plateau.

Insights IAS | InsightsonIndia

www.insightsias.com 57 www.insightsonindia.com

INS

IGH

TS

IAS

RE

VIS

ION

TE

ST

S F

OR

UP

SC

CIV

IL S

ER

VIC

ES

PR

EL

IMIN

AR

Y E

XA

M – 2

01

8

73. Which of the following statement is NOT correct.

a) The width of the Himalayan ranges decreases towards the East

b) Western Himalayas rises abruptly from the plains compared to gradual rise of Eastern

Himalayas

c) Duns are the longitudinal valley lying between lesser Himalayas and shiwaliks

d) Indus and Brahmaputra River are much older compared to Himalayas.

Answer: (b)

Justification:

The width of Himalayas in western part is around 400 km and it decreases towards to the east

which is around 125 km in Arunachal Pradesh

Because of the anti-clockwise rotation (door slam shut movement ) of Indian plate there is

abrupt rise in the heights of Eastern Himalayas whereas because of ease of pressure on western

side western Himalayas have gradual rise in height and greater width

Duns are longitudinal valleys formed as a result of folding when Eurasian plate and Indian plate

collided. They are formed between Lesser Himalayas and Shiwaliks. These valleys are deposited

with coarse alluvium brought down by Himalayan rivers.

River Indus is an antecedent river as it is considered as older than the Himalayas. The deep

gorges of the Indus, the Satluj, the Brahmaputra etc. clearly indicate that these rivers are older

than the Himalayas

74. Consider the following statements

1. Garo and Khasi hills are extensions of Purvanchals in Meghalaya forming water divide

between Brahmaputra and Barak River

2. Rajmahal hills is a denuded fold mountain around which river Ganga meanders and

moves towards the South

Which of the above statement/s is/are correct ?

a) 1 Only

b) 2 Only

c) Both 1 and 2

d) None of the above

Insights IAS | InsightsonIndia

www.insightsias.com 58 www.insightsonindia.com

INS

IGH

TS

IAS

RE

VIS

ION

TE

ST

S F

OR

UP

SC

CIV

IL S

ER

VIC

ES

PR

EL

IMIN

AR

Y E

XA

M – 2

01

8

Answer: (d)

Justification:

Garo and Khasi hills are the extensions of peninsular part of the subcontinent. Along with the

Karbi Anglong plateau , the Meghalaya plateau (comprising Garo ,Khasi and Jaintia hills ) is

separated from the Chotanagpur plateau (part of peninsular india ) by Malda fault (in Bengal).

Rajmahalhills are part peninsular plateau located in Jharkand. They are volcanic mountains

formed by Kergulean hotspot. The hills are oriented in a north-south axis with an average

elevation of 200–300 m (600-1,000 ft). The River Ganga meanders around the hills changing

direction of flow from eastwards to southwards.

75. Greater concentration of Sal and Teak forest extends over which topographic region

a) Vindhayan and Satpura ranges of Central Highland.

b) Slopes of Shiwaliks in western Himalayas

c) Purvanchals

d) Aravallis

Answer: (a)

Justification:

Sal and teak forests are best grown in central highlands. Teak and sal are tropical deciduous

trees. Madhya Pradesh has the highest concentration of teak and sal forest in India. Suitable

topographical and climatic conditions of central highlands are suitable for growth of teak and

sal.The extent of teak and sal forest is less in rest of the regions because of climatic conditions.

76. Consider the following pairs

River Merges With

1. Manjra — Krishna

2. Hiran — Narmada

3. Ramganga — Ghaghara

Which of the following is/are CORRECTLY matched

a) 2 & 3 b) 2 Only c) 1 & 2 d) 1 & 3 Only

Insights IAS | InsightsonIndia

www.insightsias.com 59 www.insightsonindia.com

INS

IGH

TS

IAS

RE

VIS

ION

TE

ST

S F

OR

UP

SC

CIV

IL S

ER

VIC

ES

PR

EL

IMIN

AR

Y E

XA

M – 2

01

8

Answer: (b)

Justification:

The Manjra is a tributary of the river Godavari. It passes through the states of Maharashtra,

Karnataka and Telangana. It originates in the Balaghat range of hills near Ahmednagar district

at an altitude of 823 metres (2,700 ft) and empties into the Godavari River.

Hiran is a right bank tributary of Narmada river. Hiran River rises in the Bhanrer range in the

Jabalpur district of Madhya Pradesh near the Kundam village at an elevation of 600 m. The river

flows in a generally south-westerly direction for a total length of 188 km to join the Narmada

from the right near Sankal village. Hiranhas the distinction of being the biggest right bank

tributary of the Narmada and drains a total area of 4,792 sq. km.

77. The irregularity in the amount of rainfall in different parts of north Indian plains is mainly

due to

a) Irregular intensity of low pressure in the north-western parts of India.

b) Difference in frequency of cyclones

c) Variation in the location of the axis of low pressure trough.

d) Variations in the amount of moisture carried by winds every year.

Answer:(c)

Justification:

The frequency of the tropical depressions originating from the Bay of Bengal varies from year to

year. Their tracks over India are mainly determined by the position of ITCZ, which is generally

termed as monsoon trough. As the axis of the monsoon trough oscillates, the tracks of these

depressions also vary. This cause wide fluctuations in the direction and the paths these

depressions take leading to variation in intensity and amount of rainfall from year to year.

Insights IAS | InsightsonIndia

www.insightsias.com 60 www.insightsonindia.com

INS

IGH

TS

IAS

RE

VIS

ION

TE

ST

S F

OR

UP

SC

CIV

IL S

ER

VIC

ES

PR

EL

IMIN

AR

Y E

XA

M – 2

01

8

78. Which of the following statement is NOT correct.

a) Variability of rainfall is highest in the regions of high rainfall.

b) Duration of south-west monsoon varies in different parts of country based on onset

and withdrawal.

c) Changes in the pressure conditions over Southern Pacific impacts the intensity of

monsoon

d) The intensity and position of high-pressure over Mascarenes affects Indian Monsoon.

Answer: (a)

Justification:

Variability of rainfall in highest in regions of low rainfall. Rainfall variability is highest in North-

western part of India.

Duration of southwest monsoons decreases as we move from southern India to northern and

north-western part of the country.

Changes in pressure and sea surface temperatures in southern pacific leads to El- nino.

High-pressure over Mascarenes along with Somali Jetstream pushes monsoon winds towards

Indian subcontinent.

79. Consider the following

1. The amount rainfall.

2. Altitude

3. Soil type

4. Humidity

The type of natural vegetation in India varies due to variation of which of the following above

factors.

a) 1 & 2 Only

b) 1,2,3

c) 1,2,4

d) All the above

Answer: (c)

Insights IAS | InsightsonIndia

www.insightsias.com 61 www.insightsonindia.com

INS

IGH

TS

IAS

RE

VIS

ION

TE

ST

S F

OR

UP

SC

CIV

IL S

ER

VIC

ES

PR

EL

IMIN

AR

Y E

XA

M – 2

01

8

Justification :

Type of vegetation in any region is not dependent on soil type. Rest of the factors determine the

kind of vegetation of a particular region.

80. Consider the following statements

1. River Mahi flows through the rift valley.

2. There are no west flowing rivers in Orissa.

Which of the above statement/s is/are correct ?

a) 1 Only

b) 2 Only

c) Both 1 and 2

d) None of the above

Answer: (a)

Justification:

The Mahi is a river in western India, it rises in Madhya Pradesh and, after flowing through the

Vagad region of Rajasthan, enters Gujarat and flows into the Arabian Sea. It is one of the rivers

in India that flows in a rift valley, the other rivers which flow through rift valley include Damodar

River in Chota Nagpur Plateau , Tapti and Narmada river.

There are many west flowing rivers in Orissa and Indravati is one of them

81. Consider the following

1. Tamil Nadu

2. Madhya Pradesh

3. Uttarakhand

Which of the above states have population density higher than National Average (2011 census)

a) 2 Only

b) 1 & 2 only

c) 1 Only

d) 2 & 3 Only

Insights IAS | InsightsonIndia

www.insightsias.com 62 www.insightsonindia.com

INS

IGH

TS

IAS

RE

VIS

ION

TE

ST

S F

OR

UP

SC

CIV

IL S

ER

VIC

ES

PR

EL

IMIN

AR

Y E

XA

M – 2

01

8

Answer: (c)

Justification :

Tamil Nadu – 555 persons per sqkm

Madhya Pradesh — 236

Uttarakhand — 189

India — 382

82. Consider the following statement

1. Population growth rate started declining after 1971.

2. As per 2011 census the sex ratio of tribal states of Chhattisgarh and Odisha is much lower

compared to national average

Which of the above statement/s is/are CORRECT ?

a) 1 Only

b) 2 Only

c) Both 1 and 2

d) None of the above

Answer : (a)

Justification:

Insights IAS | InsightsonIndia

www.insightsias.com 63 www.insightsonindia.com

INS

IGH

TS

IAS

RE

VIS

ION

TE

ST

S F

OR

UP

SC

CIV

IL S

ER

VIC

ES

PR

EL

IMIN

AR

Y E

XA

M – 2

01

8

Sex ratio of Chattisgarh – 992 and Odisha – 979 and India – 940

83. Which of the following statements is NOT correct

a) The largest part of the northern plain is formed by new alluvium deposition of Khadar

region.

b) Punjab plains on western part of northern plains is dominated by doabs.

c) Duars are the terai floodplains that lie south of Himalayan foothills

d) Alluvial plains of the Eastern India recieves higher rainfall compared to western part.

Answer: (a)

Justification :

The largest part of the northern plain is formed of older alluvium. They lie above the flood plains

of the rivers and present a terrace like feature. This part is known as Bhangar. The soil in this

region contains calcareous deposites locally known as kankar. The newer, younger deposits of

the flood plains are called khadar .

Doabs refers to the region between two rivers. Starting from east to west,

Bist Jullundhur Doab – Satluj and Beas

Bari Doab – Beas and Ravi

Rachna Doab – Rabi and Chenab

Chal/Jech Doab – Chenab and Jhelum

Sindh Sagar Doab – Jhelum and Indus

Dooars or Duars are the alluvial floodplains in northeastern India that lie south of the outer

foothills of the Himalayas and north of the Brahmaputra River basin. This region is about 30 km

(19 mi) wide and stretches over about 350 km (220 mi) from the Teesta River in West Bengal to

the Dhanshiri River in Assam. The region forms the gateway to Bhutan. It is part of the Terai-

Duar savanna and grasslands ecoregion. Duar region is rich in wildlife.

The amount of rainfall decreases from East to West.

Insights IAS | InsightsonIndia

www.insightsias.com 64 www.insightsonindia.com

INS

IGH

TS

IAS

RE

VIS

ION

TE

ST

S F

OR

UP

SC

CIV

IL S

ER

VIC

ES

PR

EL

IMIN

AR

Y E

XA

M – 2

01

8

84. Which of the following mountains chains has two dissimilar types of vegetation on its two

slopes ?

a) Western Ghats

b) Aravalis

c) Vindhayas

d) Eastern Ghats

Answer: (a)

Justification :

Because of the variation in climatic conditions the vegetation along the western and eastern

slopes are different. Western slopes have Tropical evergreen and semi-evergreen vegetation

whereas the eastern slopes are dominated by Tropical moist and dry deciduous forest.

85. Which of the following conservation sites are located in Shiwalik ranges

1. Rajaji National park

2. Valley of flowers

3. Dachigam National Park

Which of the following is/are CORRECT

a) 1 and 2 only

b) 1 only

c) 2 and 3 only

d) 1,2 and 3

Answer: (b)

Justification:

Rajaji National Park is an Indian national park and tiger reserve that encompasses the

Shivaliks, near the foothills of the Himalayas. The park is spread over 820 km2 and three districts

of Uttarakhand: Saharanpur, Dehradun and Pauri Garhwal.

Valley of flowers is located in Greater Himalayan ranges of Uttarakhand.

Insights IAS | InsightsonIndia

www.insightsias.com 65 www.insightsonindia.com

INS

IGH

TS

IAS

RE

VIS

ION

TE

ST

S F

OR

UP

SC

CIV

IL S

ER

VIC

ES

PR

EL

IMIN

AR

Y E

XA

M – 2

01

8

The Zabarwan Range is a short (20 mi (32 km) long) sub-mountain range between Pir Panjal and

Great Himalayan Range in the central part of the Kashmir Valley in the state of Jammu and

Kashmir in India. The Zabarwan mountain range possesses great Himalayan features with rich

wildlife. The Dachigam National Park, spread over 141 km2, is the main feature of the range. The

Dachigam National Park holds the last viable population of Kashmir stag (Hangul) and the largest

population of black bear in Asia.

86. Consider the following

1. Coringa

2. Pichavaram

3. Bartang Island

Which of the following is/are MANGROVE sites in India

a) 2 only

b) 1 and 3 only

c) 1 and 2 only

d) All the above

Answer: (d)

Justification:

Coringa mangroves is located in Andhra Pradesh. Coringa Wildlife sanctuary is important

conservation site of the region.

Pichavaram mangrove wetland is in Tamil Nadu

One of the biggest mangrove forest is found in Bartang island in Andamans.

87. Consider the Endangered species and their conservation sites of India

1. Nilgiri Tahr — Eravikulam National Park

2. Lion Tailed Macaque — Papikonda National park

3. One-horned Rhinoceros — Orang National park

Which of the following is/are CORRECTLY matched

a) 3 only

b) 2 and 3 only

c) 1 and 3 only

d) 1 and 2 only

Insights IAS | InsightsonIndia

www.insightsias.com 66 www.insightsonindia.com

INS

IGH

TS

IAS

RE

VIS

ION

TE

ST

S F

OR

UP

SC

CIV

IL S

ER

VIC

ES

PR

EL

IMIN

AR

Y E

XA

M – 2

01

8

Answer: (c)

Justification:

Lion Tailed Macaque is endangered species of Western Ghats .Papikonda National park is

in East Godavari district of Andhra Pradesh.

Nilgiri Tahr is endemic to western ghats found in greater numbers in Eravikulam National

Park in Kerala

Orang national park is in Assam known for conservation of one-horned Rhinoceros.

88. Identify the only conservation site of Lakshadweep Islands

a) Mount Harriett National Park

b) Pitti Bird sanctuary

c) Mahatma Gandhi Marine National park

d) Malvan Marine wildlife sanctuary

Answer: (b)

Justification:

Pitti Bird Sanctuary is a small reef that is approximately 24 north-west of Kavaratti, Pitti is an

important nesting place for pelagic birds such as the sooty tern (Sterna fuliginosa), the greater

crested tern (Sterna bergii) and the brown noddy (Anous stolidus).

89. Alpine shrub and meadows extend over which following states in north-east

a) Mizoram and Tripura

b) Nagaland and Tripura

c) Sikkim and Arunachal

d) Assam and Meghalaya

Answer: (c)

Insights IAS | InsightsonIndia

www.insightsias.com 67 www.insightsonindia.com

INS

IGH

TS

IAS

RE

VIS

ION

TE

ST

S F

OR

UP

SC

CIV

IL S

ER

VIC

ES

PR

EL

IMIN

AR

Y E

XA

M – 2

01

8

Justification:

Eastern Himalayas extend over the states of Sikkim and Arunachal. The elevation extends

beyond 6000mts where Alpine shrub and meadows are found.

90. Consider the following

National Park River

1. Corbett National park — Alaknanda

2. Silent Valley National Park — Kaveri

Which of the following is/are CORRECTLY matched

a) 1 only

b) 2 only

c) Both 1 and 2

d) None of the above

Answer: (d)

Justification:

River Ramganga flows through Corbett National park

Small river Kunthipuzha flows through Silent Valley National Park.

91. The first Pelican Bird Festival-2018 was recently organised at

a) Pulicat lake

b) Kolleru lake

c) Ashtamudi wetland

d) Vembanad-Kol wetland

Solution: b.

It was organised by the AP State Tourism Department.

http://www.thehindu.com/todays-paper/tp-national/tp-andhrapradesh/pelican-festival-at-

kolleru-a-huge-hit/article22653092.ece

Insights IAS | InsightsonIndia

www.insightsias.com 68 www.insightsonindia.com

INS

IGH

TS

IAS

RE

VIS

ION

TE

ST

S F

OR

UP

SC

CIV

IL S

ER

VIC

ES

PR

EL

IMIN

AR

Y E

XA

M – 2

01

8

92. Consider the following pairs:

Launch Pad – Location

1. Kennedy Space Centre – San Francisco, California

2. Europe’s Spaceport – Kourou, French Guiana

3. Satish Dhawan Space Centre – Abdul Kalam Island, Odisha

4. Rocket Lab Launch Complex – Mahia Peninsula, New Zealand

Which of the pairs given above is/are correctly matched ?

a) 1, 2 and 4 only

b) 1 and 3 only

c) 2 and 4 only

d) 1, 2, 3 and 4

Solution: c.

You can eliminate the 1st given pair easily. Because satellites and missile tests always are

launched from the east coast (http://www.thehindu.com/sci-tech/science/why-are-all-

satellites-and-missiles-launched-from-the-east-coast/article7202159.ece)

Kennedy Space Centre – Florida

Europe’s Spaceport – Kourou, French Guiana

Satish Dhawan Space Centre – Sriharikota, Andhra Pradesh

Rocket Lab Launch Complex – Mahia Peninsula, NZ

Improvisation (passing reference to the launch pad at Kourou): http://www.thehindu.com/sci-

tech/science/isro-needs-4-years-to-catch-up-with-satellite-demand-dr-

sivan/article22651083.ece

93. Consider the following statements:

1. Mangrove forests only grow at tropical and subtropical latitudes

2. The Sundarbans lies in latitudes north of the Tropic of Cancer

Select the correct answer using the code given below:

a) 1 only

b) 2 only

c) Both 1 and 2

d) Neither 1 nor 2

Insights IAS | InsightsonIndia

www.insightsias.com 69 www.insightsonindia.com

INS

IGH

TS

IAS

RE

VIS

ION

TE

ST

S F

OR

UP

SC

CIV

IL S

ER

VIC

ES

PR

EL

IMIN

AR

Y E

XA

M – 2

01

8

Solution: a.

Mangrove trees grow in areas with low-oxygen soil, where slow-moving waters allow fine

sediments to accumulate. Mangrove forests only grow at tropical and subtropical latitudes near

the equator because they cannot withstand freezing temperatures.

The Sundarbans lies south of the Tropic of Cancer.

https://oceanservice.noaa.gov/facts/mangroves.html;

Improvisation: http://www.thehindu.com/news/national/other-states/sundarbans-

mangroves-struggle-to-find-new-ground/article22785340.ece;

94. Consider the following statements:

1. The Purchasing Managers’ Index (PMI) can be used as an indicator of the economic health

of the manufacturing sector only

2. While the Index of Industrial Production (IIP) is an output measure, the PMI is an

indication of the activity at the input, or purchasing level

Which of the statements given above is/are correct ?

a) 1 only

b) 2 only

c) Both 1 and 2

d) Neither 1 nor 2

Solution: b.

In the news (http://www.thehindu.com/business/Economy/what-is-the-lowdown-on-

industrial-renewal/article22785218.ece): “Industrial activity, as measured by the government’s

Index of Industrial Production (IIP) and the private sector Purchasing Managers’ Index (PMI),

has improved significantly over the last two months.. Growth in the IIP soared to 8.8% in

November, the highest since October 2015, and stood at a slightly slower but still robust 7.07%

in December.. Similarly, the PMI surged to a 60-month high of 54.7 in December and came in at

52.4 in January (The PMI is usually released at the start of the month, much before most of the

official data on industrial output, manufacturing and GDP growth becomes available. It is,

therefore, considered a good leading indicator of economic activity). These two sets of data are

interesting because not only do they show the picture from both the government and private

sector sides but also highlight different elements of the sectors they measure. While the IIP is an

output measure, the PMI is an indication of the activity at the input, or purchasing, level. If both

show strong growth, the implication is an overall recovery in industrial activity and sentiments.”

Insights IAS | InsightsonIndia

www.insightsias.com 70 www.insightsonindia.com

INS

IGH

TS

IAS

RE

VIS

ION

TE

ST

S F

OR

UP

SC

CIV

IL S

ER

VIC

ES

PR

EL

IMIN

AR

Y E

XA

M – 2

01

8

PMI or a Purchasing Managers’ Index (PMI) is an indicator of business activity – it can be

utilised as an indicator for both, the manufacturing and services sectors. It is a survey-based

measures that asks the respondents about changes in their perception of some key business

variables from the month before.

The PMI is derived from a series of qualitative questions. Executives from a reasonably big

sample, running into hundreds of firms, are asked whether key indicators such as output, new

orders, business expectations and employment were stronger than the month before and are

asked to rate them. A figure above 50 denotes expansion in business activity. Anything below 50

denotes contraction. Higher the difference from this mid-point, greater the expansion or

contraction.

https://economictimes.indiatimes.com/news/economy/indicators/what-is-purchasing-

managers-index-pmi/articleshow/6259031.cms;

https://www.investopedia.com/terms/p/pmi.asp

95. Which among the following initiatives by NITI Aayog has a direct bearing on competitive

federalism ?

1. Sustainable Action for Transforming Health

2. Samavesh

3. Champions of Change

Select the correct answer using the code given below:

a) 1 only

b) 1 and 2

c) 1 and 3

d) 2 and 3

Solution: a.

SATH (http://pib.nic.in/newsite/PrintRelease.aspx?relid=165545): NITI Aayog has launched

SATH, a program providing ‘Sustainable Action for Transforming Human capital’ with the

State Governments. The vision of the program is to initiate transformation in the education and

health sectors. SATH aims to identify and build three future ‘role model’ states each for health

and education systems (NITI Aayog has selected Uttar Pradesh, Assam, and Karnataka to

improve healthcare delivery and key outcomes. In Education, Madhya Pradesh, Odisha, and

Jharkhand have been selected for support to better learning outcomes.)

Samavesh (http://niti.gov.in/content/concept) is a programme launched by the NITI Aayog to

link together various lead Knowledge and Research Institutions.

Insights IAS | InsightsonIndia

www.insightsias.com 71 www.insightsonindia.com

INS

IGH

TS

IAS

RE

VIS

ION

TE

ST

S F

OR

UP

SC

CIV

IL S

ER

VIC

ES

PR

EL

IMIN

AR

Y E

XA

M – 2

01

8

Champions of Change (http://niti.gov.in/content/champions-change-transforming-india-

through-g2b-partnership) is an initiative organised by NITI Aayog to “transform India through

G2B (Government-to-Business) partnership”.

NITI Aayog and Competitive federalism

Introduction of Digital Transformation Index and Innovation Index for ranking states

Launch of the Health Index, which is to become an annual exercise across the country

evaluating health outcomes.

SATH

School Education Quality Index (SEQI)

Composite Water Management Index

http://pib.nic.in/newsite/PrintRelease.aspx?relid=176698